англ іспит(повний)

Додано: 13 грудня 2023
Предмет:
366 запитань
Запитання 1

A specimen presents an endocrine system organ covered with capsule made of connective tissue. Septa branch off from the capsule inwards and divide the organ into lobules. Each lobule consists of two cell types: neurosecretory pinealocytes (polygonal cells with processes) located in the center and gliacytes (astrocytes) located at the periphery. What organ is represented in this specimen?

варіанти відповідей

Adrenal medulla

Thyroid gland

Pituitary gland

Hypothalamus

Epiphysis

Запитання 2

A diver that submerged to the depth of 75 meters detected signs of CNS functional disturbance: excitation, lapse of concentration, euphoria leading to professional errors. What substance has toxic effect on the neurons, thus leading to the development of these signs?

варіанти відповідей

Ammonia

Nitrogen

Lactate

Carbon dioxide

Oxygen

Запитання 3

A person in the state of nervous tension develops transverse wrinkles on the forehead. What muscle contracts to produce this result?

варіанти відповідей

M. procerus

 M. corrugator supercilii

M. auricularis anterior

M. occipitofrontalis

M. temporoparietalis

Запитання 4

Autopsy of a man with tuberculosis has revealed a 3x2 cm large cavity in the superior lobe of the right lung. The cavity was interconnected with a bronchus, its wall was dense and consisted of three layers: the internal layer was pyogenic, the middle layer was made of tuberculous granulation tissue and the external one was made of connective tissue. What is the most likely diagnosis?

варіанти відповідей

Acute focal tuberculosis

Fibrous focal tuberculosis

Tuberculoma

 Acute cavernous tuberculosis

Fibrous cavemous tuberculosis

Запитання 5

A patient diagnosed with botulism has been prescribed antibotulinic serum for treatment. What immunity will be formed in the given patient?

варіанти відповідей

Antitoxic passive immunity

Antimicrobic active immunity

Antitoxic active immunity

Antimicrobic passive immunity

 Infection immunity

Запитання 6

Roentgenologically confirmed obstruction of common bile duct resulted in preventing bile from inflowing to the duodenum. What process is likely to be disturbed?

варіанти відповідей

Salivation inhibition

Hydrochloric acid secretion in stomach

Fat emmulgation

Protein absorption

 Carbohydrate hydrolysis

Запитання 7

A patient with a nasal trauma presents with skull fracture that circles the piriform opening. What bone is damaged?

варіанти відповідей

Maxillary

 Lacrimal

 Ethmoid

Frontal

Nasal

Запитання 8

Auscultation reveals that in the patient's intercostal space along the parasternal line on the right the II heart sound is better heard than the I heart sound. What valve produces this sound when closing?

варіанти відповідей

Tricuspid valve

Semilunar pulmonary valve

Semilunar aortic valve

Bicuspid valve

Bicuspid and tricuspid valves

Запитання 9

Micromanipulator was used to extract one of the two centrioles of centrosome (cell center) fromhepatocyte (liver cell). What process will STOP OCCURRING in this cell?

варіанти відповідей

Glycogen synthesis

 Protein biosynthesis

Division

Lipid synthesis

 Energy exchange

Запитання 10

An 84-year-old patient suffers from parkinsonism One of the pathogenetic development elements of this disease is deficiency of a certain mediator in some of the brain structures. Name this mediator

варіанти відповідей

Dopamine

Noradrenaline

Adrenaline

Histamine

Acetylcholine

Запитання 11

To test teeth sensitivity they are sprayed with cold or hot water. What structure of cerebral cortex provides subjective estimation of this thermal test?

варіанти відповідей

Precentral gyrus

First temporal convolution

Posterior central gyrus

Central fissure

Middle frontal gyrus

Запитання 12

 Oxidative decarboxylation of pyruvic acid is catalyzed by a multienzyme complex with several functionally linked coenzymes. Name this complex:

варіанти відповідей

Coenzyme A (COASH), flavin adenine dinucleotide (FAD), pyridoxal-5-phosphate, tetrahydrofolic acid, carnitine 

Flavin adenine dinucleotide (FAD), tetrahydrofolic acid, pyridoxal-5-phosphate, thymidine diphosphate (TDP), choline

 Lipoic acid, tetrahydrofolic acid, pyridoxal-5-phosphate, methylcobalamin

Nicotine amide adenine dinucleotide (NAD), pyridoxal-5-phosphate, thymidine diphosphate (TDP), methylcobalamin, biotin

Thymidine diphosphate (TDP), flavin adenine dinucleotide (FAD), coenzyme A (COASH),

nicotine amide adenine dinucleotide (NAD), lipoic acid

Запитання 13

A 55-year-old man was diagnosed with acute glomerulonephritis. Name the main mechanism of anemia development in this case:

варіанти відповідей

Decreased glomerular filtration

Decreased tubular reabsorption

Decreased erythropoietin synthesis

Decreased synthesis of renal prostaglandins

Renal azotemia

Запитання 14

Erythrocytes of the patient with hemolytic anemia present with significant decrease of pyruvate kinase activity. What methabolic process is disturbed in this case?

варіанти відповідей

Glycolysis

Gluconeogenesis

Pentose-phosphate pathway of glucose

oxidation

Glycogenolysis

Glycogen synthesis

Запитання 15

In dental practice atropine sulfate is used to decrease salivation. This drug belongs to the following group:

варіанти відповідей

Muscle relaxants

a-adrenergic agonists

Ganglionic blockers

Muscarinic antagonists

B-adrenoceptor antagonists

Запитання 16

On microscopic examination of leftovers of the canned meat eaten by a patient with severe food toxicoinfection the following was detected: grampositive bacilli with subterminal staining defect and configuration alteration of bacilli generally resembling a tennis racket. What agent was detected?

варіанти відповідей

E. coli

S. enteritidis

S. Aureus

C. botulinum

P. Vulgaris

Запитання 17

A patient with displaced fracture of the

right coronoid process of mandible has been delivered to a first-aid center. What muscle had displaced the coronoid process?

варіанти відповідей

Right lateral pterygoid muscle

Right temporal muscle

Left masticatory muscle

Right medial pterygoid muscle

Right masticatory muscle

Запитання 18

A patient with parodontosis was prescribed a fat-soluble vitamin that actively participates in redox processes in the organism. This antioxidant is a growth factor, has antixerophthalmic action, and contributes to maintenance of normal vision. In dental practice it is used to accelerate mucosal re-epithelization during parodontosis. Name this substance:

варіанти відповідей

Ergocalciferol

Menadione (Vicasolum)

Cyanocobalamin

Tocopherol acetate

Retinol acetate

Запитання 19

Cells of sensory spinal ganglions are a part of reflex arches. What type of neurons are these cells?

варіанти відповідей

-

Bipolar

Pseudounipolar

Multipolar

Unipolar

Запитання 20

To take a sample of cerebrospinal fluid for analysis, a doctor makes a puncture into subarachnoid space. To prevent damage to the spinal cord, the needle must be inserted between the two following vertebrae

варіанти відповідей

IV and V thoracic

III and IV lumbar

 I and II humbar

XII thoracic and I lumbar

XI and XII thoracic

Запитання 21

A woman, who had undergone mastectomy due to breast cancer, was prescribed a course of radiation therapy. What vitamin preparation has marked antiradiation effect due to its antioxidant activity?

варіанти відповідей

Folic acid

Ergocalciferol

Riboflavin

Tocopherol acetate

Cyanocobalamin

Запитання 22

A child presents with a wound behind the mastoid bone. Bright red blood streams from the wound. Damage was sustained to the branches of the following artery:

варіанти відповідей

A. maxillaris

A temporalis superior

A. occipitalis

A. carotis externa

A. carotis interna

Запитання 23

During dehelmintization a patient expelled long fragments of a segmented helminth. In some segments their width exceeds their length; there is a rosettelike uterus in the segment center. Name the helminth:

варіанти відповідей

Hymenolepis nana

Diphylobotrium latum

Taeniarhynchus saginatus

Taenia solium

Echinococcus

Запитання 24

A patient suffering from ciliary arrhythmia with anamnesis of bronchial asthma should be prescribed an antiarrhythmic drug. What antiarrhythmic drug is CONTRAINDICATED in this case?

варіанти відповідей

Verapamil

Anaprilin (Propranolol)

Novocainamide (Procainamide)

Ajmaline

Nifedipine

Запитання 25

In the process of embryogenesis of dentomaxillary system there was an enamel disturbance detected What source of tooth development is damaged in this case?

варіанти відповідей

 Dental bulb

Mesoderm

Mesenchyme

Dental saccule

Oral cavity epithelium

Запитання 26

Chronic overdosage of glucocorticoids leads to the development of hyperglycemia. What process of carbohydrate metabolism is responsible for this effect?

варіанти відповідей

Aerobic glycolisis

Gluconeogenesis

Pentose-phosphate cycle

Glycogenolysis

Glycogenesis

Запитання 27

A patient is in the state of hypoglycemic coma. What hormone can cause this condition if overdosed?

варіанти відповідей

Somatotropin

Cortisol

Corticotropin

Insulin

Progesterone

Запитання 28

According to the data collected by WHO researchers, every year there are approximately 250 millon malaria cases occur in the world. This disease can be encountered predominantly in tropical and subtropical areas. The spread of this disease matches the natural habitat of the following genus of mosquitoes:

варіанти відповідей

Culiseta

Anopheles

Mansonia

Aedes

Culex

Запитання 29

During examination the doctor performs auscultation to assess the functioning of the patient's mitral valve. Where can the sound of this valve be auscultated?

варіанти відповідей

At the edge of the sternum in the 2nd intercostal space on the rightd intercostal space on ther" ht intercostal space on the left

At the edge of the sternum over the 5th costal cartilage on the left

At the apex of the heart

 At the edge of the sternum over the 5th costal cartilage on the right

At the edge of the sternum in the nd intercostal space on the right

Запитання 30

An experimental animal, a dog, received a weak solution of hydrochloric acid through a tube inserted into the duodenum. Primarily it will result in increased secretion of the following hormone:

варіанти відповідей

Secretin

Gastrin

Cholecystokinin

Neurotensin

Histamine

Запитання 31

On microscopic examination of a surgical biopsy material (part of the lip with an ulcer) near the ulcer margins and under the ulcer floor in the connective tissue of mucosa there are epithelial complexes composed of atypical stratified epithelium with pathological mitotic figures. Within these complexes there are accumulations of bright pink concentric formations. What pathology is it?

варіанти відповідей

Papilloma

Basal cell carcinoma

Transitional cell carcinoma

Squamous cell nonkeratinous carcinoma

Squamous cell keratinous carcinoma

Запитання 32

 A man, who for a long time has been suffering from chronic mandibular osteomyelitis, died of chronic kidney disease. Autopsy revealed large lardaceous kidneys. What process had occurred in the kidneys?

варіанти відповідей

Contracted kidney

 Necrotic nephrosis

Renal amyloidosis

Arterial nephroscleros

 Glomerulonephritis

Запитання 33

A trauma patient has wound in the temporal region, with trickle of bright-red blood streaming from it. What blood vessel is damaged?

варіанти відповідей

 A. auricularis posterior

A. temporalis superficialis

A. facialis

A. occipitalis

A. maxillaris

Запитання 34

X-ray detected pus accumulation in the sphenoidal simis. The pus is being excreted into the following nasal meatus:

варіанти відповідей

Right inferior nasal meatus

Right and left superior nasal meatus

Left inferior nasal meatus

Right middle nasal meatus

Left middle nasal meatus

Запитання 35

A woman presents with edemas. In her urine there is a large amount of protein excreted What nephron segment is functionally disturbed in this case?

варіанти відповідей

 Distal convoluted tubule

Renal corpuscle

Descending limb of loop of Henle

Proximal convoluted tubule

Ascending limb of loop of Henle

Запитання 36

During examination of the oral cavity at the vestibular surface of the lower right incisor there was detected a rounded growth on the thin pedicle. Histologically in the connective tissue there are numerous thin-walled sinusoids, hemorrhage areas, hemosiderin foci, and giant cells resembling osteoclasts. Make the diagnosis:

варіанти відповідей

Gramular cell ameloblastoma

Gingival fibromatosis

 Cavernous hemangioma

Giant cell epulis

Angiomatous epulis

Запитання 37

 A patient presents with disturbed patency of the airways at the level of small and medium bronchial rubes. What acidbase imbalance can the patient develop?

варіанти відповідей

Metabolic acidosis

Respiratory alkalosis

Metabolic alkalosis

Acid-base balance remains unchanged

 Respiratory acidosis

Запитання 38

A child with signs of rickets has been prescribed a certain liposoluble vitamin drug by the pediatrician and dentist. This drug affects the metabolism of phosphorus and calcium in the body and facilitates calcium accumulation in bone tissue and dentine. If its content in the body is insufficient, there develop disorders of ossification process, dental structure, and occlusion. Name this drug:

варіанти відповідей

Menadione (Vicasolum)

Thyroidin

Retinol acetate

Ergocalciferol

Tocopherol acetate

Запитання 39

 A patient is diagnosed with deformed posterior portion of the nasal septum. What bone is deformed?

варіанти відповідей

Vertical plate of palatine bone

Vomer

Medial pterygoid plate

 Perpendicular plate of ethmoid bone

 Lateral pterygoid plate

Запитання 40

A patient has been administered conduction anesthesia with novocaine in preparation for tooth extraction. After the anesthesia administration the patient developed swelling and hyperemy around the injection site, skin itch, general fatigue, motor agitation. Name the developed complication:

варіанти відповідей

Idiosyncrasy

Drug dependence

Allergy

Inflammation

Tachyphylaxis

Запитання 41

A patient presents with dysfunction of the cerebral cortex accompanied by epileptic seizures. He has been administered a biogenic amine synthetized from glutamate and responsible for central inhibition. What substance is it?

варіанти відповідей

Acetylcholine

y-aminobutyric acid

Histamine

Serotonin

Dopamine

Запитання 42

Beriberi is a classical example of thiamine deficiency. Active form of this vitamin is synthesized by an enzyme belonging to the following group:

варіанти відповідей

Transferases

Lyases

Oxidoreductases

Isomerase

Hydrolases

Запитання 43

 A patient came to the doctor with complaints of general weakness and sleep disturbances. Objectively the patient's skin is yellow. In blood there is increased concentration of direct bilirubin and bile acids. Acholic stool is observed. What condition can be characterized by these changes?

варіанти відповідей

 Familial nonhemolytic (Gilbert's) syndrome

 Hemolytic jaundice

Mechanical jaundice

Parenchymatous jaundice

Chronic cholecystitis

Запитання 44

A specimen of intestine demonstrates complex branching tubuloalveolar glands with their ends embedded in the submucous layer. What organ is it?

варіанти відповідей

 Jejunum

Ileum

Duodenum

Cecum

Colon

Запитання 45

Often the cause of secondary immmunodeficiency is an infectious affection of an organism, when agents reproduce directly in the cells of immune system and destroy them. Specify the diseases, during which the described above occurs

варіанти відповідей

Tuberculosis, mycobacteriosis

Infectious mononucleosis, AIDS

Dysentery, cholera

Poliomyelitis, viral hepatitis type A

Q fever, typhus

Запитання 46

Autopsy of a 58-year-old man, who had been suffering from rheumatic heart disease and died of cardiopulmonary decompensation, revealed gray diffuse film- and fiber-shaped coating in his pericardium. What type of inflammation is characteristic of this pericarditis?

варіанти відповідей

Hemorrhagic

 Suppurative

Serous

Diphtheritic fibrinous

Croupous fibrinous

Запитання 47

A 72-year-old man with hepatocirrhosis developed hepatic coma. Its development is caused by the substances, that are being neutralized in the liver, entering into general circulation through portacaval shunts (portal hypertension syndrome) and necrosis of hepatic cells. What type of hepatic coma is characterized by these presentations?

варіанти відповідей

Mixed

Parenchymatous

Hepatocellular

Ketoacidotic

Shunt

Запитання 48

A doctor has established significant growth retardation, disproportional body build, and mental deficiency of a child. What is the most likely cause of this pathology?

варіанти відповідей

Hypopituitarism

Insufficient nutrition

Hyperthyroidism

Genetic defects

Hypothyroidism

Запитання 49

Alcoholic intoxication is accompanied by disturbed motor coordination and equilibrium due to the damage caused to structural elements of the cerebellum. Functional disturbance of the following cells occurs in the first place:


варіанти відповідей

Pyriform cells

Stellate cells

Granule cells

Basket cells

Fusiform cells

Запитання 50

A woman complains of painful chewing, especilly when she moves her lower jaw backwards. What muscles are affected?

варіанти відповідей

Medial prerygoid muscles

Lateral pterygoid muscles

Posterior bundles of the temporal muscles

Masseter muscles

Anterior bundles of the temporal muscles

Запитання 51

 A patient suffers from diabetes mellitus with fasting hyperglycemia over 7.2 mmol/L. What blood plasma protein would allow to assess the patient's glycemia level retrospectively (4-8 weeks prior to examination)?

варіанти відповідей

Fibrinogen

Ceruloplasmin

Albumin

C-reactive protein

Glycated hemoglobin

Запитання 52

A patient with acute pancreatitis presents with significantly increased urine diastase content. What proteolysis inhibitor must be included into complex therapy of this patient?

варіанти відповідей

Festal

Digestal

Pancreatine

Mezym forte

Contrykal (Aprotinin)

Запитання 53

Carious cavities of a 29-year-old patient contain parasitic protozoa. It is established that they belong to the Sarcodina class. Specify these protozoa:

варіанти відповідей

Entamoeba gingivalis

Entamoeba histolutica

Lamblia intestinalis

 Amoeba proteus

Entamoeba coli

Запитання 54

Histological investigation of an extracted tooth revealed presence of necrotic detritus with fatty acid crystals and numerous microbe colonies in the pulp chamber. What diagnosis is the most likely?

варіанти відповідей

Complicated chronic pulpitis

Purulent pulpitis

Chronic pulpitis

Serous pulpitis

Pulp gangrene

Запитання 55

Bacilli were extracted from the investigated sample. The bacilli are curved, extremely mobile, gramnegative, form no spores or capsules, have anaerobic form of respiration. They form transparent smooth colonies in alkaline agar, ferment saccharose and mannose into acid, produce exotoxin, fibrinolysin, collagenase, hyaluronidase. What agent was extracted?

варіанти відповідей

Comma bacillus

Proteus

 Dysentery bacillus

Blue pus bacillus

Colibacillus

Запитання 56

Histologic specimen of a tooth slice shows a tissue consisting of intercellular substance permeated with tubules, in which cellular processes of odontoblasts are situated. What tissue is presented in this histologic specimen?

варіанти відповідей

Pulp

Dentin

Periodontium

Cement

Enamel

Запитання 57

In the course of experiment it is necessary to detect muscle excitation. For this purpose the following measurement should be made:

варіанти відповідей

Contraction strength

Ion concentration

Contraction duration

Electromyogram

Mechanomyogram

Запитання 58

A. 68-year-old man, who had been suffering from essential hypertension for a long time, was delivered to a resuscitation unit with hemiplegia. The patient died after 7 hours. On autopsy: in the right cerebral hemisphere there is a cavity 5x5 cm in size with uneven margins, filled with dark red blood clots. What cerebral circulation disorder developed in the patient?

варіанти відповідей

Thrombosis

Petechial hemorrhage

 Hemorrhagic infiltration

Local venous hyperemia

Hematoma

Запитання 59

A 50-year-old man, who has been suffering for a long time from viral hepatitis, developed mental impairments, impairments consciousness, and motor disturbances (tremor, ataxia, etc.). What is the mechanism of such condition?

варіанти відповідей

Decreased synthesis of albumins and globulins in the liver

Disturbed lipid exchange in the liver

Decreased detoxification function of the liver

Insufficient phagocytic function of stellate macrophages

Alterations in the lipid composition of blood

Запитання 60

A 46-year-old patient consulted an oculist about drooping of the upper eyelid. On examination he was diagnosed with a brain tumor. The pathological process must have affected the nuclei of the following pair of cranial nerves:

варіанти відповідей

VI

IV

|||

||

VII

Запитання 61

A patient has been diagnosed with bicuspid valve insufficiency. Where is this valve located?

варіанти відповідей

Between the left atrium and left ventricle

Between the right atrium and right ventricle

Between the left and right ventricles

At the aortic orifice

Between the left and right atria

Запитання 62

A lancelet embryo is at the developmental stage during which its cells multiply, while its general volume remains practically unchanged. What developmental stage is it?

варіанти відповідей

Histogenesis

 Neurulation

Gastrulation

Cleavage

Organogenesis

Запитання 63

A patient consulted a doctor about an increased pain sensitivity of the ear sirin and ear canal. Palpation behind the sternocleidomastoid muscle was painful. Such clinical presentations are typical of the irritation of the following nerve:

варіанти відповідей

N. vagus

N. auricularis magnus

Nn. supraclaviculares

N. occipitalis minor

N. transversus colli

Запитання 64

A victim of an earthquake has been remaining under debris for 7 days without food or water. What type of starvation is it?

варіанти відповідей

Quantitative

Qualitative

Complete

Complete with continued hydration

Incomplete

Запитання 65

A patient suffers from disturbed blood


supply of superior lateral surface of the cerebral


hemispheres. What blood vessel is damaged?

варіанти відповідей

Posterior communicating artery

Medial cerebral artery

Anterior cerebral artery

Anterior communicating artery

Posterior cerebral artery

Запитання 66

To treat osteomyelitis, a patient was prescribed an antibiotic that easily penetrates into bone tissue. Name this drug:

варіанти відповідей

Lincomycin hydrochloride

Streptomycin sulfate

Amphotericin B

Polymyxin B

Cefazolin

Запитання 67

After the cerebral hemorrhage a patient

developed aphasia lost the ability to articulate words. The hemorrhage is localized in the:

варіанти відповідей

Inferior frontal gyrus

 First temporal convolution

Second temporal convolution


Middle frontal gyrus

Superior frontal gyrus

Запитання 68

Microscopic examination of pus sample taken from mandibular fistula canal and stained by Gram's method has revealed druses with gram-positive coloring in the center and cone-shaped structures with gram-negative coloring. Such morphology is characteristic of the agent of:

варіанти відповідей

Actinomycosis

Candidiasis

Staphylococcal osteomyelitis

Anaerobic infection

Fusobacteriosis

Запитання 69

Analysis of sputum taken from a patient with suspected pneumonia revealed slightly elongated gram-positive diplococci with tapered opposite ends. What microorganisms were revealedin the sputum?

варіанти відповідей

Staphylococcus aureus

Streptococcus pneumoniae

 Neisseria meningitidis

Neisseria gonorrhoeae

Klebsiella pneumonise

Запитання 70

During examination a dentist noticed bowl-shaped defects on the frontal surface of the patient's canines in the cervical area. Microscopically the pulp is covered with compacted dentin, slightly atrophied and sclerosed. What pathological process occurs in the patient?

варіанти відповідей

Stage of chalky white spots

Cuneiform defects

Superficial caries

Median caries

Deep caries

Запитання 71

A. 35-year-old man has come to a dentist with complaints of decreased density of the dental tissue and increased brittleness of his teeth during consumption of hard food Laboratory analysis measured Ca/P correlation in the enamel sample. What value of Ca/P indicates increased demineralization?

варіанти відповідей

1.85

0.9

1.67

2.5

1.5

Запитання 72

Heterozygous parents with A (II) and B (III) blood groups according to ABO system gave birth to a child. What is the probability of the child having 0 (1) blood group?

варіанти відповідей

0.25

0

0.75

0.5

1

Запитання 73

. A patient presents with osteoporosis; hypercalcemia and hypophosphatemia are observed in the patient's blood. What is the cause of this condition?

варіанти відповідей

Increased thyroxin secretion

 Increased corticosteroid secretion

Inhibited corticosteroid secretion

Inhibited parathormone secretion

Increased parathormone secretion

Запитання 74

 Autopsy of a 45-year-old man revealed the following: wrinkled shagreen-resembling intima in the ascending aorta; elastic staining is negative. Wall of the left ventricle is up to 1,5 cm in width; no myocardial alterations. Specify the most probable disease of the aorta:

варіанти відповідей

Rheumatic vasculitis

Atherosclerosis of the aorta

-

Aortic failure

Syphilitic mesaortitis

Запитання 75

A patient with autoimmune thyroiditis has been prescribed a peptide hormonal agent. Specify this agent:

варіанти відповідей

L-thyroxin

riquilar

Tamoxifen

Trimethoprim

Запитання 76

Breakdown of cyclic monophosphate (CAMP) and cyclic guanosine monophosphate (cGMP) into simple, non-cyclic nucleoside monophosphates is catalyzed by the following enzyme:

варіанти відповідей

Glycogen phosphorylase

Phosphodiesterase

Protein kinase

Adenylate cyclase

Glucose 6-phosphatase

Запитання 77

During development of a frostbite the exposed skin becomes pale and its temperature drops. What mechanism is the basis of these developments?

варіанти відповідей

Dermal and subcutaneous vasodilatation

Visceral vasoconstriction

Closure of arteriovenous anastomoses

Decreased heart rate

Reflex vasoconstriction

Запитання 78

A dental patient was prescribed a psychosedative for his fear of pain. What drug would be the most effective in this case?

варіанти відповідей

Lithium carbonate

Aminazine

Valerian tincture

Sodium bromide

Diazepam

Запитання 79

Electric current has affected skeletal muscle fiber resulting in depolarization of the membrane. Depolarization develops due to the following ions penetrating the membrane

варіанти відповідей

CL-

HC

Nа+

C

Запитання 80

A patient in a grave condition has been delivered to an admission ward. Examination revealed pupil mydriasis, no reaction to the light, considerable reddening and dryness of skin and mucous membranes. What drug could have caused the intoxication symptoms?

варіанти відповідей

Adrenalin hydrochloride

Proserin

Atropine sulphate

Dithylimum

Pilocarpine hydrochloride

Запитання 81

A 30-year-old woman complains of intense thirst and dryness of the mouth that developed after a severe emotional shock. Laboratory analysis revealed increase of the patient's blood sugar level up to 10 mmol/L. What endocrine gland is affected in the patient?

варіанти відповідей

Adrenal glands

Gonads

Pancreas

Thyroid gland

Pineal gland

Запитання 82

In the course of experiment the vagus nerve of the test animal was severed, which resulted in the animal developing constant tachycardia. What effect of parasympathetic nervous system on cardiac performance is demonstrated by this experiment?

варіанти відповідей

Inhibition

Paradoxical response

Stimulus summation

Stimulation

 Mixed effect

Запитання 83

A histological specimen shows terminal secretory parts of glands made of conic cells with basophilic cytoplasm and a roundish nucleus in the centre. Specify the type of terminal secretory parts by the type of secretion:

варіанти відповідей

Serous

Sebaceous

Seromucous

mucous

Combined

Запитання 84

A patient with dislocated jaw was given a short-acting muscle relaxant by a doctoName this drug:

варіанти відповідей

Cytitonum (Cytisine)

Dithylinum (Suxamethonium chloride)

Procaine

Pyridostigmine hydrobromide

Papaverine hydrochloride

Запитання 85

 During autopsy of the patient, who died of cardiovascular insufficiency, the patient's right foot is darkly colored. The vessels of the patient's thigh. are partially obstructed by grayish-red clots. On the vessel walls there are yellowish-gray spots and fibrous plaques, some of which are of stony density. What clinicopathological type of atherosclerosis was complicated in the patient?

варіанти відповідей

Cerebral atherosclerosis

Atherosclerosis of lower extremities

Renal atherosclerosis

Atherosclerosis of aorta

Vascular intestinal atherosclerosis

Запитання 86

Biological material taken from a patient contains several species of microorganisms (staphylococci and streptococci) that are causative agents of the patient's disease. Name this type of infection

варіанти відповідей

Mixed infection

Consecutive infection

Superinfection

Coinfection

Reinfection

Запитання 87

During experiment, the myotome mas destroyed in the rabbit fetus. This manipulation will result in malformation of the following structure:

варіанти відповідей

Axial skeleton

Dermal connective tissue

Serous membranes

Skeletal muscles

Smooth muscles

Запитання 88

A 33-year-old man has a cystic growth connected to the 2nd molar of the lower jaw. Within the cystic cavity there is a rudimentary tooth. On microscopy: inner cystic surface is covered with stratified squamous epithelium, there are groups of mucin-producing cells. What diagnosis is the most likely?

варіанти відповідей

Primordial cyst

Follicular cyst

Follicular ameloblastoma

Radicular cyst

Periodontitis

Запитання 89

A woman with essential hypertension has been hospitalized. The patient presents with aneurysm of a. communicans posterior of the cerebrum arterial circle. What vessels of arterial circle are normally joined with this artery?

варіанти відповідей

A. carotis interna et a. cerebri media

A. cerebri anterior et a. cerebri media

A. cerebri media et a. cerebri posterior

A. carotis intema et a. cerebri posterior

A. carotis extema et a. cerebri anterior

Запитання 90

A patient complains of frequent bowel movements and stool with blood admixtures ("raspberry jelly" stool). Microscopic examination revealed large mononuclear cells with absorbed red blood cells. What protozoon is this morphological structure typical of?

варіанти відповідей

Campylobacter jejuni

Entamoeba histolytica

Giardia lamblia

Balantidium coli

Toxoplasma gondii

Запитання 91

When determining comparative tissue radiosensitivity, it was revealed that different tissues have different level of sensitivity toward ionizing radiation. What tissue of those listed below is the most radiosensitive?

варіанти відповідей

Hematopoietic

Muscular

Cartilaginous

Bone

Nerve

Запитання 92

A patient is diagnosed with pancreatitis. Starch decomposition disturbance occurs in the patient's intestine due to deficiency of the following pancreatic enzyme:

варіанти відповідей

Carboxypeptidase

Lipase

Chymotrypsin

Amylase

Tripsin

Запитання 93

A 45-year-old man with a history of left-sided croupous pneumonia died of multiple traumas received as the result of a car accident. On autopsy in the lower lobe of his left lung its posterolateral wall is attached to the chest wall with fibrous adhesions. The lobe is diminished, dense, fleshy on section, grayish-pink in color; its pieces sink, when placed in water. Histological analysis reveals diffuse excessive growth of fibrous connective tissue in these areas. Name this complication of croupous pneumonia:

варіанти відповідей

Abscess

Emphysema

Caneous degeneration

Atelectasis

Gangrene

Запитання 94

Due to sustained trauma the patient presents with unevenly dilated pupils (anisocoria). What muscle is blocked?

варіанти відповідей

Musculus ciliaris

Musculus rectus inferior

Musculus sphincter pupillae

Musculus rectus lateralis

Musculus rectus superior

Запитання 95

According to the law of constancy of chromosome mumbers, most animal species have definite and constant chromosome mumber. The mechanism that maintains this constancy during sexual reproduction of organisms is called:

варіанти відповідей

Schizogony

Amitosis

Regeneration

Meiosis

-

Запитання 96

 A patient suffering from acute bronchitis with difficult expectoration was prescribed acetylcysteine. What drug action will provide curative effect?

варіанти відповідей

Alkalinization of sputum

Stimulation of the bronchial glands

Mucoproteins depolymerization

Reflex stimulation of bronchiolar peristalsis

Activation of bronchial ciliated epithelium

Запитання 97

A 26-year-old woman presents with skin rashes and itching after eating citrus fruits. Prescribe her a drug that is an Hihistamine receptor antagonist.

варіанти відповідей

Acetylsalicylic acid

Menadione (Vicasolum)

Paracetamol

Dimedrol (Diphenhydramine)

Analgin (Metamizole)

Запитання 98

During inspection of dental tools for sterility in one case gram-positive cocci were detected. They were situated in clusters and yielded positive plasma coagulation reaction; the cocci were fermenting mannitol in anaerobic conditions and exhibiting lecithinase activity. What microorganism was detected?

варіанти відповідей

Staph. aureus

Corinebacterium xerosis

Str. pyogenes

St. epidermidis

St. saprophiticus

Запитання 99

A patient has undergone recurring blood tests that revealed sharp fluctuations of glucose content: significant increase in absorptive state and significant decrease in postabsorptive state. What

pathology can be the cause of it?

варіанти відповідей

Diabetes mellitus type I

 Diabetes mellitus type II

Acromegaly

Endemic goiter

Aglycogenosis (glycogenosis type 0)

Запитання 100

A connective tissue specimen stained with hematoxylin-eosin shows isogenous cell groups surrounded with basophilic intercellular substance.

No fibrous structures are detected. What type of connective tissue is it?

варіанти відповідей

Loose fibrous tissue

Elastic cartilage tissue

Dense fibrous tissue

Splenial bone tissue

Hyaline cartilage tissue

Запитання 101

In 8 days after a surgery the patient developed tetanus. The surgeon suspects this condition to be caused by suture material contaminated by tetanus agent. The material is delivered to a bacteriological laboratory. What nutrient medium is required for primary inoculation of the suture material?

варіанти відповідей

Sabouraud agar

Hiss medium

Egg-yolk salt agar

Kitt-Tarozzi medium

 Endo agar

Запитання 102

After restoration of maxllary incisors with artificial crowns a 44-year-old woman was found to have a brownish growth in the form of a node 15 mm in diameter. Histological study revealed that under the stratified squamous epithelium of the gingiva there was a connective tissue mass with numerous sinusoidal vessels, oval-shaped monomuclear cells forming osteoid substance, and polynuclear giant cells that destroyed the alveolar ridge of the upper jaw. What is the most likely diagnosis?

варіанти відповідей

Giant cell epulis

Angiomatous epulis

Fibromatous epulis

Eosinophilic granuloma

 Gingival fibromatosis

Запитання 103

A 67-year-old man was delivered to a cardiology department with complaints of periodical pains in his heart, dyspnea caused by even slight exertion, cyanosis and edemas. ECG shows additional excitations of heart ventricles. Name this type of rhythm disturbance:

варіанти відповідей

Tachycardia

Fibrillation

Bradycardia

Flutter

Extrasystole

Запитання 104

As a result of an injury a child developed an abscess of adipose tissue of the cheek. With time the process spread to the lateral surface of the pharynx. Pus had spread along the following fascia:

варіанти відповідей

Temporal

-

Masticatory

Parotid

Bucco-pharyngeal

Запитання 105

After spinal trauma the patient presents with absence of voluntary movements and tendon reflexes; sensitivity is retained only in the lower extremities. What is the mechanism of these disturbances and what part of the spine was injured?

варіанти відповідей

Central paralysis, coccyx

-

Peripheral paralysis, cervical spine

Spinal shock, thoracic spine

Spinal shock, cervical spine

Запитання 106

A victim of an accident has hemorrhage in the area of lateral surface of the mastoid process. What branch of the external carotid artery supplies this area with blood?

варіанти відповідей

A. facialis

A. pharyngea ascendens

A. temporalis superficialis

A. maxillaris

A. auricularis posterior

Запитання 107

After examining a patient a doctor recommended him to exclude rich meat and vegetable broths, spices, and smoked products from the diet, since the patient was found to have:

варіанти відповідей

Reduced motility of the gastrointestinal tract

Increased secretion of hydrochloric acid by the stomach glands

Reduced secretion of hydrochloric acid by the stomach glands

Reduced salivation

 Biliary dyskinesia

Запитання 108

Preventative examination of a 55year-old patient revealed type II diabetes mellitus. An endocrinologist detected an increase in body weight and liver enlargement. The man is a non-smoker and does not abuse alcohol but likes to have a "hearty meal". Histological examination by means of diagnostic liver puncture revealed that the hepatocytes were enlarged mostly on the lobule periphery, their cytoplasm had transparent vacuoles that reacted positively with sudan III. What liver pathology was revealed?

варіанти відповідей

Acute viral hepatitis

Alcohol hepatitis

Portal liver cirrhosis

Chronic viral hepatitis

Fatty hepatosis

Запитання 109

On autopsy of a 69-year-old woman, who for a long time had been suffering from hypertension, the pathologist determined that both of her kidneys are dense, markedly diminished with finegrained surface. These changes are indicative of:

варіанти відповідей

Atrophy due to inadequate blood supply

Dysfunctional atrophy

Hypoplasia

Compression atrophy

Senile renal atrophy

Запитання 110

A 30-year-old man had suffered a thoracic trauma in a traffic accident, which resulted in disturbance of extemal respiration. What ventilatory failure can be observed in this case?

варіанти відповідей

Dysregulatory

Extrapulmonary restrictive

Mixed type

Obstructive

Pulmonary restrictive

Запитання 111

In Western Europe nearly half of all congenital malformations occur in the children conceived in the period, when pesticides were used extensively in the region. Those congenital conditions result from the following influence:

варіанти відповідей

Carcinogenic

Malignization

Mutagenic

Teratogenic

Mechanical

Запитання 112

During a brain surgery stimulation of the cerebral cortex resulted in tactile and thermal sensations in the patient. What gynus was stimulated?

варіанти відповідей

Precentral gyrus

Postcentral gyrus

Cingulate convolution

Parahippocampal gyrus

Superior temporal gyrus

Запитання 113

After an abortion a 25-year-old woman developed oliguria, anuria, and increasing azotemia. The patient died of acute renal failure. Autopsy revealed degeneration and necrosis of the convoluted renal tubules epithelium. What disease was the cause of death in this case?

варіанти відповідей

Rapidly progressive glomerulonephritis

Chronic glomerulonephritis

Renal amyloidosis

Acute necrotizing nephrosis

Acute pyelonephritis

Запитання 114

It is known that in metabolism of catecholamine mediators the special role belongs to monoamine oxidase (MAO). How does this enzyme activate these mediators (noradrenaline, adrenaline, dopamine)?

варіанти відповідей

Carboxylation

Methyl group removal

Amino group attachment

Hydrolysis

Oxidative deaminization

Запитання 115

A student, who unexpectedly met his girlfriend, developed an increase in systemic arterial pressure. This pressure change was caused by the intensified realization of the following reflexes:

варіанти відповідей

Unconditional parasympathetic

Conditional sympathetic

Unconditional sympathetic

Conditional sympathetic and parasympathetic

Conditional parasympathetic

Запитання 116

A patient with acne is prescribed doxycycline hydrochloride. What should the patient be warned against, regarding administration of this drug?

варіанти відповідей

Take with large amount of liquid, preferably milk

 Do not take with vitamin preparations

Course of treatment should not exceed 1 day

Avoid prolonged exposure to the sun

Take before eating

Запитання 117

A 60-year-old with hepatocirrhosis developed hemorrhagic syndrome. What mechanism leads to the development of this condition?

варіанти відповідей

 Increased portal venous pressure

 Deceased blood oncotic pressure

Emergence of neurotoxins in the blood

Decreased synthesis of prothrombin and fibrinogen

Reduction of hepatic glycogen stores

Запитання 118

In postabsorptive state glycogen synthesis is increased in liver and muscles. The syntesis involves the following substance:

варіанти відповідей

Adenosine triphosphate (ATP)

Thymidine triphosphate (TTP)

Uridine triphosphate (UTP)

Cytidine triphosphate (CTP)

 Guanosine triphosphate (GTP)

Запитання 119

During experiment the processes of food and water hydrolysis products absorption were studied. It was determined that these processes mainly occur in the following gastrointestinal segment:

варіанти відповідей

Rectum

Small intestine

Oral cavity

Large intestine

Stomach

Запитання 120

A newborn failed to take his first breath. Autopsy revealed that despite unobstructed airways the lungs of the newborn were unable to stretch. What is the most likely cause of this condition?

варіанти відповідей

Pleural thickening

Bronchial rupture

Alveolar enlargement

Absence of surfactant

Bronchial narrowing

Запитання 121

A 55-year-old man with acute heart failure has been administered a quickrelief cardiac glycoside. Which of the following drugs has been given to the patient?

варіанти відповідей

Digitoxin

Strophanthin

Milrinone

Celanid

Adonisidum

Запитання 122

On histological examination of biopsy material taken from the liver of a woman, who for a long time had been suffering from viral hepatitis type B, the pathologist detected diffuse hepatic fibrosis with formation of porto-portal and portocentral fibrotic septa and disturbance of the liver lobular structure (development of pseudolobules). What process can be characterized by the given morphological changes?

варіанти відповідей

Acute hepatitis

Hepatocellular carcinoma

Cholestasis

Hepatic cinhosis

Chronic hepatitis

Запитання 123

What coenzyme of flavin-dependent dehydrogenases participates in the reactions of tricarboxylic acid cycle?

варіанти відповідей

Thymidine diphosphate (TDP)

Flavin mononucleotide (FMN)

Flavin adenine dinucleotide (FAD)

Heme

Nicotinamide-adenine dinucleotide (NAD+)

Запитання 124

Red bone marrow has been damaged

under radioactive emission of 5 Gy. What determines the red bone marrow sensitivity towards ionizing radiation?

варіанти відповідей

High content of peroxides in the cells

Intensive cell division

Radiosensitizers in the cells

High content of free radicals in the cells

Destructive effect of radiotoxins on DNA

synthesis

Запитання 125

Gastroscopy of a patient revealed the lack of mucus in the coating of the mucous membrane. This can be caused by the dysfunction of the following cells of the gastric wall:

варіанти відповідей

Main exocrinocytes

Parietal cells of gastric glands

Endocrinocytes

Cervical cells

Cells of prismatic glandular epithelium

Запитання 126

 A patient has a history of chronic heart failure. Which of the following hemodynamic parameters is a major symptom of cardiac decompensation development?

варіанти відповідей

Decreased stroke volume

Tonogenic dilatation

Increased central venous pressure

Increased peripheral vascular resistance

Tachycardia development

Запитання 127

A bacteriological laboratory received a sample of dried fish from an outbreak of food poisoning. Inoculation of the sample on Kitt-Tarozzi medium revealed microorganisms resembling tennis racket. These microorganisms are causative agents of the following disease:

варіанти відповідей

Botulism

Salmonellosis

Dysentery

Typhoid fever

Diphtheria

Запитання 128

Autopsy of a 7-year-old child, who died of uncompensated congenital heart disease, revealed increase in mass and volume of the thymus. On microscopy thymus structure is normal. What pathologic process had occurred in the thymus?

варіанти відповідей

Thymic agenesis

Thymoma

Thymic dysplasia

Congenital thymomegaly

Accidental involution

Запитання 129

The patient exhausted by starvation presents with intensification of the following process in the liver and kidneys:

варіанти відповідей

Urea synthesis

Uric acid synthesis

Hippuric acid synthesis

Bilirubin synthesis

Gluconeogenesis

Запитання 130

On tooth section in the area of the root apex there is a tissue consisting of cells with processes surrounded by mineralized intercellular substance. Name this tissue:

варіанти відповідей

Enamel

Cellular cement

Reticulofibrous bone tissue

Periodontium

Mantle dentin

Запитання 131

At the end of winter a student, who had been lately in the state of nervous tension, developed a case of URTI after overexposure to cold. What is the case of this disease?

варіанти відповідей

Overexposure to cold

Improper diet

Pathogenic agent

Hypovitaminosis

Nervous stress

Запитання 132

Histologic specimen of endometrium

demonstrates isolated epithelial cells with

chromosomes that form a "plate" located in the equatorial plane of the cell. What stage of the cell cycle is it?

варіанти відповідей

Telophase

Anaphase

Interphase

Metaphase

Prophase

Запитання 133

A patient suffering from acute vascular purpura 15 prescribed a firstgeneration antihistamine with local anaesthetic, antispasmodic, and sedative action. Specify this drug:

варіанти відповідей

Dibazol (Bendazol)

Dithylin

Diazolin (Mebhydrolin)

Dimedrol (Diphenhydramine)

Droperidol

Запитання 134

A 3-year-old child presents with facial deformation that was gradually developing over the course of 6 months and manifests as symmetrical enlargement of both mandibular angles. Microscopy shows the space between the bone trabeculae to be filled with connective tissue that contains mumerous blood vessels and smaller primitive bone trabeculae. What disease is the most likely in this case?

варіанти відповідей

Cherubism

Eosinophilic granuloma

Osteosarcoma

Giant-cell tumor of the bone

Fibroma

Запитання 135

A woman had been taking synthetic hormones during her pregnancy. Her newborn girl presents with excessive hairiness which has formal resemblance to adrenogenital syndrome. This sign of variability is called:

варіанти відповідей

Replication

Recombination

Mutation

Heterosis

Phenocopy

Запитання 136

 A patient complains of painful chewing, especially when his lower jaw moves forward and to the side. It indicates functional disorder of the following muscles:

варіанти відповідей

Masseter muscles

Lateral pterygoid muscles

 Medial pterygoid muscles

Mylohyoid muscles

Temporal muscles

Запитання 137

During cell analysis, their cytoplasm was determined to have high content of aminoscyl tRNA synthetase. This enzyme ensures the following process:

варіанти відповідей

 Repair

Replication

Amino acid activation

Elongation

Transcription

Запитання 138

A patient came to the traumatologist with complaints of developing difficulties during active extension of elbow. What muscle is the most likely to be damaged?

варіанти відповідей

M. deltoideus

M. triceps brachii

M. pectoralis minor

M. coracobrachialis

M. latissimus dorsi

Запитання 139

To terminate hypertensive crisis the patient was administered solution of magnesium sulfate. What route of drug administration should be chosen?

варіанти відповідей

Duodenal

Rectal

Intravenous

Oral

Intra-arterial

Запитання 140

A patient with glossitis presents with disappearance of lingual papillae, reddening and burning pain in the tongue. Blood test: erythrocytes 2,210 12/1, hemoglobin- 103 g/l, color index - 1.4.

What type of anemia is it?

варіанти відповідей

B 12 folate-deficiency 12 folate-deficient

Iron refractory

a-thalassemia

B-thalassemia

 Iron deficiency

Запитання 141

A young man has been performing physical exercises, holding a weight for a long time. What kind of muscle contraction is the most characteristic of these exercises?

варіанти відповідей

Isotonic

 Asynchronous

Isometric

Single

Isovolumetric

Запитання 142

A patient with damaged muscles of the lower limbs has been delivered to a firstaid center. What cells enable reparative regeneration of muscle fibers and restoration of muscle function?

варіанти відповідей

 Myosatellitocytes

Endotheliocytes

Adipocytes

Fibroblasts

Plasmocytes

Запитання 143

Among organic substances of a cell there is a polymer composed of dozens, hundreds, and thousands of monomers. This molecule is capable of self-reproduction and can be an information carrier. X-ray structure analysis shows this molecule to consist of two complementary spiral threads. Name this compound:

варіанти відповідей

Carbohydrate

RNA

DNA

Hormone

Cellulose

Запитання 144

 A pregnant woman developed severe toxemia with exhausting recurrent vomiting throughout a day. By the end of the day she developed tetanic convulsions and dehydration. The described changes were caused by the following type of acid-base imbalance:

варіанти відповідей

Nongaseous metabolic acidosis

Gaseous acidosis

Nongaseous excretory acidosis

Gaseous alkalosis

Nongaseous excretory alkalosis

Запитання 145

Autopsy of a man who died of ethylene glycol poisoning revealed that his kidneys are slightly enlarged, edematic; their capsule can be easily removed. Cortical substance is broad and light gray. Medullary substance is dark red. What pathology did this man develop?

варіанти відповідей

Acute glomerulonephritis

Acute pyelonephritis

 Lipoid nephrosis

Acute tubular-interstitial nephritis

Necrotic nephrosis

Запитання 146

A 24-year-old patient has been administered glutamic acid to treat epilepsy. Medicinal effect in this case occurs not due to glutamate itself, but due to the product of its decarboxylation:

варіанти відповідей

y-aminobutyric acid

Histamine 4-monooxygenase

 Dopamine

Taurine

Serotonin

Запитання 147

A patient was diagnosed with a monogenic hereditary disease. Name this disease:

варіанти відповідей

Poliomyelitis

Hymenolepiasis

Peptic ulcer disease of the stomach

Hypertension

Hemophilia

Запитання 148

During oral cavity examination a dentist noticed eruption of the permanent canines in a child. The child grows and develops normally. Determine the child's age:

варіанти відповідей

 6-7 years

8-9 years

13-16 years

11-13 years

9-10 years

Запитання 149

. A 19-year-old young man has been examined in a nephrological hospital. Increased potassium content was detected in secondary urine of the patient. Such alterations are the most likely to be caused by the increased secretion of the following hormone:

варіанти відповідей

Oxytocin

Aldosterone

Glucagon

Adrenaline

Testosterone

Запитання 150

A woman is diagnosed with Turner's syndrome (karyotype 45, X0). How many autosomal pairs would her somatic cells contain?

варіанти відповідей

44

22

45

24

23

Запитання 151

Autopsy of a 46-year-old man, who had untreated enteric infection and died of sepsis, revealed the following: perirectal phlegmon, multiple ulcers of the rectum and sigmoid colon some of which are perforated; mucosa of these intestinal segments is thickened and covered with firmly attached grayish films. What is the most likely disease in this case?

варіанти відповідей

Cholera

Amebiasis

Tuberculosis

Dysentery

Typhoid fever

Запитання 152

Electron diffraction pattern of a spermatozoon clearly presents a sheathlike structure localized in the spermiocalyptrotheca and participating in dissolution of ovular membranes. Name this structure:

варіанти відповідей

Segmented columns

Acrosome

Centriole

Microtubules

Axoneme

Запитання 153

During gastrulation an embryo proceeds from histiotrophic to hematotrophic nutrition. What provisory organ makes it possible at first?

варіанти відповідей

Chorion

Amnion

Trophoblast

Allantois

Yolk sac

Запитання 154

Autopsy of a 52-year-old man revealed changes in his lungs: there is a segmented area of caseous necrosis in the upper right lung, the segments merge with each other. The lung is enlarged, dense, yellowishcolored on section, there are fibrinous films on the pleura. Name the type of tuberculosis:

варіанти відповідей

Cirrhotic tuberculosis

Caseous pneumonia

Tuberculoma

Infiltrative tuberculosis

Acute cavernous tuberculosis

Запитання 155

A patient has suffered a head injury. On examination there is a subcutaneous hematoma in the temporal area. What vessel was damaged, thus resulting in the hematoma development?

варіанти відповідей

A. buccalis

A. maxillaris

A. temporalis superficialis

A. auricularis posterior

A. Occipitalis

Запитання 156

What enzyme has demineralization effect, i. e. intensifies decomposition of mineral components of the tooth tissues?

варіанти відповідей

Glucose 6-phosphatase

Phosphotransferase

Acid phosphatase

Glycogen phosphorylase

Alkaline phosphatase

Запитання 157

The patient's examination in a hospital specialised in diseases of nervous system has revealed absence of light-induced miosis. It is caused by the damage of the following brain structures:

варіанти відповідей

Red nuclei of mesencephalon

Reticular nuclei of medulla oblongata

Vegetative nuclei of the 3rd pair of cranial nerves

Reticular nuclei of mesencephalon

Hypothalamus nuclei

Запитання 158

Differentiation of B-lymphocytes into

plasma cells leads to synthesis of immunoglobulins that ensure specific immune response of the body. Differentiation of B-lymphocytes takes place in the following organ of immune system:

варіанти відповідей

Liver

Tоnsils

Red bone marrow

Thymus

Thyroid gland

Запитання 159

 A child with a foreign body in the lungs has been hospitalized. What bronchus is the most likely to contain this foreign body, if its diameter approximates to 1,5 cm?

варіанти відповідей

Left segmental bronchus

Right segmental bronchus

Right primary bronchus

Lobar bronchus

Left primary bronchus

Запитання 160

Examination of histological specimen of oral mucosa reveals non-keratinized stratified squamous epithelium with lymphocyte infiltrations. What structure of oral cavity is the most likely to be represented by this mucosa

варіанти відповідей

Hard palate

Lip

Cheek

Tonsil

Gums

Запитання 161

 It is necessary to perform urinary bladder catheterization of an adult man. Resistance to the catheter can occur in the following structure or part of the uretlira:

варіанти відповідей

Membranous part

External urethral orifice

Spongiose part

Internal urethral orifice

Prostatic part

Запитання 162

 During thermal stimulation it is characteristic of oral cavity blood vessels to:

варіанти відповідей

Present no response towards thermal stimuli 

Respond with constriction to hot stimuli

Respond with constriction to cold stimuli

Dilate in response to both cold and hot stimuli

Respond depending on the vessel functional

condition

Запитання 163

 A 38-year-old woman developed an attack of bronchial asthma. What bronchial spasmolytic for emergency medical aid is a beta-2-adrenergic agonist?

варіанти відповідей

Atropine

 Ipratropium bromide

Salbutamol

Adrenaline

Platyphyllin

Запитання 164

A patient with electrical injury to the neck area developed pathologic fixed sideways flexion of the head towards the injured area, while the face is fixed away from the injury. What neck muscle sustained scarring?

варіанти відповідей

Omohyoid muscle

Anterior scalene muscle

Sternocleidomastoid muscle

Digastric muscle

Trapezius muscle

Запитання 165

 Due to morbid affection of the supraoptic and paraventricular nuclei of the hypothalamus a 40-year-old patient has developed polyuria (10-12 liters per day) and polydipsia. The following hormone is deficient, thus leading to this disturbance:

варіанти відповідей

Vasopressin

Thyrotropin

Somatotropin

Oxytocin

Corticotropin

Запитання 166

30 minutes after dental treatment the patient developed red itching spots on the face and oral mucosa. The patient was diagnosed with urticaria. What bioactive substance with vasodilating and pruriginous effect is produced during this type of allergic reaction?

варіанти відповідей

Prostaglandin E2

Bradykinin

Histamine

Interleukin-1

Leukotriene B4

Запитання 167

A slide mount of an ovary presents a rounded structure with glandular cells that contain lipid droplets. Name this structure:

варіанти відповідей

Primary ovarian follicle

 Primordial ovarian follicle

Corpus luteum

Mature ovarian follicle

Corpus albicans

Запитання 168

A doctor has made a diagnosis of gingivitis and recommended the patient to rinse the oral cavity with an oxidizing agent. Specify this agent:

варіанти відповідей

Phenol

Salicylic acid

Hydrogen peroxide

Boric acid

Brilliant green

Запитання 169

Autopsy of a 72-year-old man with recurrent transmural myocardial infarction revealed his epicardium and pericardium membranes to be swollen, thickened, coarce, as if covered in hair. Name the type of inflammation that occurred in the cardiac membranes:

варіанти відповідей

 Suppurative

Serous

Diphtheritic

Croupous

Catarrhal

Запитання 170

Typical signs of food poisoning caused by C. botulinum include diplopia, swallowing and respiration disorders. These signs develop due to

варіанти відповідей

Adhesion of the agent to enterocyte receptors

Enterotoxin action

Enterotoxic shock development

Adenylate cyclase activation

Exotoxin action

Запитання 171

Increased stumulation rate of isolated

heart of a rabbit leads to incomplete relaxation of the heart ventricles due to:

варіанти відповідей

Increased sodium content in cardiomyocytes

Inhibition of K-Na pump

Calcium accumulation in cardiomyocytes

Increased potassium content in cardiomyocytes

Increased potassium content in the interstitial tissue

Запитання 172

To determine functional state of the patient's liver, the analysis of animal indican excreted with urine was conducted. This substance is produced in the process of detoxification of putrefaction products of a certain amino acid, which takes place in the large intestine. Name this amino acid:

варіанти відповідей

Valine

Glycine

Tryptophan

Cysteine

Serine

Запитання 173

During examination a neurologist taps the tendon under the patient's kneecap with a reflex hammer to evaluate reflex extension of the Inee. This response is provoked by stimulation of the following receptors:

варіанти відповідей

Muscle spindles

Tactile receptors

Golgi tendon organ

Articular receptors

Nociceptors

Запитання 174

Amino acids join to each other in ribosomes of granular endoplasmic reticulum. Knowing the sequence of amino acids and applying genetic code, it is possible to determine the sequence of nucleoids in

варіанти відповідей

rRNA

Carbohydrates

Proteins

Introns

mRNA

Запитання 175

To drain the oral cavity a dentist places a tampon between the cheek and the 2nd upper molar. This way secretion of the following salivary gland WILL NOT be able to accumulate in the oral cavity:

варіанти відповідей

Sublingual gland

Labial glands

Submandibular gland

Parotid gland

Lingual gland

Запитання 176

After introdiction of adrenaline the patient's blood glucose level increased. It is caused by intensified:

варіанти відповідей

Glycogenolysis in the liver

Glycogenolysis in the muscles

 Glycolysis in the skeletal muscles

Glycogen synthesis

Glycolysis in the liver

Запитання 177

A victim of a traffic accident has lost thoracic respiration but retaines diaphragmal. The spinal cord is most likely to be damaged at:

варіанти відповідей

VI-VII cervical segments

 I-II cervical segments

I-II lumbar segments

|-II sacral segments

XI-XII cervical segments

Запитання 178

 During examination of the patient's oral cavity a dentist noticed deformation of the teeth and a crescent indentation on the upper right inscisor. The teeth are undersized, barrel-shaped tooth cervix is wider than its edge. The patient uses a hearing aid, suffers from visual impairment. What type of syphilis affects teeth in such a way?

варіанти відповідей

Secondary

 Neurosyphilis

Late congenital

Primary

Early congenital

Запитання 179

 A surgeon accidentally damaged a nerve that innervates mylohyoid muscle. Name this nerve:

варіанти відповідей

N. accessories

N. hypoglossus

N. trigeminus

N. facialis

N. glossopharyngeus

Запитання 180

A puncture sample has been taken from the inguinal lymph nodes of a patient provisionally diagnosed with plague. The sample was inoculated into hard nutrient medium. What shape will the colonies have, if the diagnosis is confirmed?

варіанти відповідей

"Lace handkerchief"

 "Shagreen leather"

"Lion's mane"

"Mercury drops"

"Dewdrops"

Запитання 181

 Along with normal hemoglobin types there can be pathological ones in the body of an adult. Specify one of them:

варіанти відповідей

HbAl

HbO2

HBA2

HbS

HbF

Запитання 182

 A patient complains of acute spastic abdominal pain, frequent urge to defecate, liquid bloody feces with mucus. Laboratory analysis of fecal smear revealed inconstant in shape organisms with erythrocytes. What is the most likely diagnosis?

варіанти відповідей

Lambliasis

Schistosomiasis

Balantidiasis

Amebiasis

Intestinal trichomoniasis

Запитання 183

 Ulcer disease of the duodenum has been detected in a 38-year-old man. A treatment was prescribed after which the patient considered himself to be healthy. However, half a year later the patient developed pain in the epigastrium, heartburn, and insomnia. The patient's condition can be estimated as a:

варіанти відповідей

Remission

Development of chronic disease

 Latent period

Pathological state

Relapse

Запитання 184

 A microspecimen of heart shows rectangular cells from 50 to 120 micrometer in size with central position of nucleus and developed myofibrils. The cells are connected by intercalated discs. These cells are responsible for the following function

варіанти відповідей

Endocrine

Protective

Function of heart contractions

Function of impulse conduction

Regeneratory

Запитання 185

To treat tuberculosis, an antibiotic that colors urine red is prescribed. Name this antibiotic:

варіанти відповідей

Amoxicillin

Rifampicin

Cefotaxime

Erythromycin

Nitroxoline

Запитання 186

 Examination revealed the patient to have decreased secretory function of the nasal cavity glands. What nerve provides parasympathetic innervation of these glands?

варіанти відповідей

N. petrosus minor

N. maxillaris

N. chorda tympani

N. petrosus major

N. petrosus profundus

Запитання 187

During teeth examination on the lateral surface of the first upper molar there was detected a cone-shaped carious cavity with base oriented towards the tooth surface and apex - towards the tooth center. Softened dentin is visible at the floor of the carious cavity. Make the diagnosis:

варіанти відповідей

-

Dentin caries

 Enamel caries

Cement caries

Tooth erosion

Запитання 188

A 67-year-old patient has ordered a full fiunctional denture. It was necessary to extract his left upper canine. After infraorbital anaesthesia the patient presented with progressing hematoma in the frontal part of his face. The patient was found to have an injury of the artery that is the branch of

варіанти відповідей

A. temporalis superficialis

A maxillaris

A. ophthalmica

A. facialis

A. labialis superior

Запитання 189

A patient has been hospitalised with provisional diagnosis of diphyllobothriasis. What food products can be the cause of this condition?

варіанти відповідей

Pork

Beef

Milk and eggs

Fish

Запитання 190

 A forensic laboratory received clothes of a citizen, who a day before was reported missing. The clothes were found in a shed, there are red stains identified as blood by an expert. What reaction should be performed to determine whether these red stains are dried human blood?

варіанти відповідей

Circular precipitation

 Flocculation

Complement binding

Agglutination

Enzyme immunoassay

Запитання 191

 In the armpits of a patient there are small (1-1,5 mm), dorsoventrally flattened, wingless, blood-sucking insects. Their larvae have been developing in the armpits as well. What disease is caused by these insects?

варіанти відповідей

Sleeping sickness

 Chagas' disease

Relapsing fever

Phthiriasis

Plague

Запитання 192

A patient consulted a therapeutist with complaints of pain in the chest, cough, fever. X-ray revealed eosinophilic infiltrates in the lungs, further investigation of which allowed to detect presence of larvae. What helminthiasis is it characteristic of

варіанти відповідей

Cysticercosis

Ascariasis

Echinococcosis

Fascioliasis

Trichiniasis

Запитання 193

Various types of muscle contractions occurring in the alimentary canal of a test animal were studied and their different functional purposes were determined. It was noted that only one type of motor activity occurred in the circular and longitudinal muscles. Name this motor activity:

варіанти відповідей

Peristalsis

 Mastication

Nonpropulsive segmental activity

Pendular movements of intestine

Tonic contraction of sphincters

Запитання 194

The patient, who for a long time has been keeping to an unbalanced lowprotein diet, developed fatty liver infiltration. Name the substance, absence of which in the diet can lead to this condition:

варіанти відповідей

Cholesterol

 Arachidonic acid

Biotin

Alanine

Methionine

Запитання 195

 After a traffic accident a man presents with severe blood loss, consciousness disturbance, low blood pressure, as well as compensatory activation of the reninangiotensin system, which results in.

варіанти відповідей

 Intensification of heart contractions

Increased blood coagulation

Hyperproduction of vasopressin

Intensification of erythropoiesis

Hyperproduction of aldosterone

Запитання 196

Examination of a patient revealed glycosuria and hyperglycemia. He complains of dry mouth, itchy skin, frequent urination, thirst. He has been diagnosed with diabetes mellitus. What is the cause of polyuria in this patient?

варіанти відповідей

Increased plasma oncotic pressure

Increased filtration pressure

Increased urine osmotic pressure

Decreased cardiac output

Decreased plasma oncotic pressure

Запитання 197

Gastrulation is the period of embryogenesis, when germinal layers are formed, resulting in three-layered structure of an embryo. What gastrulation mechanism is characteristic of a human embryo?

варіанти відповідей

Delamination and immigration

Invagination

 Immigration and invagination

Epiboly

Delamination and epiboly

Запитання 198

A man came to a surgeon with complains of pain in his lower right abdomen. On deep palpation the doctor detected a painful area in the right inguinal region. What part of the intestine is the most likely to be affected with pathological process?

варіанти відповідей

Sigmoid colon

Transverse colon

Rectum

Descending colon

Cecum

Запитання 199

A man presents with suppurative wound in the area of mastoid bone, which resulted in development of cerebral meningitis in the patient. Specify the way of infection penetration into the patient's cranial cavity:

варіанти відповідей

V. emissariae mastoidea

V. facialis

V.v. tympanicae

V. auricularis

 V.v. labirinthi

Запитання 200

A 28-year-old patient complains of frequent gingival hemorrhages. Blood test revealed the clotting factor II (prothrombin) deficiency.

What phase of blood coagulation is impaired in this

patient?

варіанти відповідей

Clot retraction

Thrombin generation

Vascular-platelet haemostasis

Fibrinolysis

-

Запитання 201

The autopsy of the body of a man, who suffered from toxic diphtheria and died on the 9th day from the onset of disease with signs of cardiac decompensation, has revealed the following: dilated cardiac cavities, dull flaccid myocardium, myocardial section is variegated. Microscopy revealed fatty degeneration of cardiac hystiocytes, large foci of myocytolysis, stromal edema with scant lymphocytic and macrophageal infiltration.

Specify the type of myocarditis in this case: 

варіанти відповідей

Focal intermediate exudative

Septic

 Interstitial

Gramulomatous

Alterative

Запитання 202

The autopsy of a 37-year-old man has revealed the following: in the sorta on the smooth glossy ivory-colored Intima there are yellowish-gray spots blending with each other, which form stripes that do not protrude from the intima surface. Microscopy reveals swelling and destruction of elastic membranes, diffuse impregnation of aortic wall with orange granules (if stained with Sudan III), orange coloring of macrophage cytoplasm and nonstriated muscle elements. Specify this process

варіанти відповідей

Liposclerosis of the aorta

Atherocalcinosis of the sorta

Lipoidosis of the aorta

Atheromatosis of the aorta

Imbibition of the aorta with cadmium salts

Запитання 203

An accident had resulted in a 65-year-old man drowning in a lake. Resuscitation measures allowed to restore his respiration and cardiac function. What factor prolongs the period of apparent death

варіанти відповідей

Elderly age

Hypothermia

-

Hyperthermia

Prolonged preagony and agony

Запитання 204

 A 45-year-old man came to the hospital complaining of sensory loss in the posterior 1/3 of his tongue. Which pair of the cranial nerves is functionally disturbed?

варіанти відповідей

XII

IX

V

VIII

X

Запитання 205

After facial trauma the patient developed a buccal hematoma. What salivatory gland has its outflow blocked by the hematoma?

варіанти відповідей

Submandibular

Parotid

Buccal

Sublingual

Lingual

Запитання 206

Blood serum of the patient has milky appearance. Biochemical analysis revealed high content of triacylglycerols and chylomicrons. This condition is caused by hereditary defect of the following enzyme:

варіанти відповідей

Adipose tissue hormone-sensitive lipase

 Pancreatic lipase

Phospholipase

Phosphodiesterase

Lipoprotein lipase

Запитання 207

A 30-year-old woman has decreased enzyme content in the pancreatic juice. This condition can be caused by insufficient secretion of the following hormone:

варіанти відповідей

Secretin

Cholecystokinin-pancreozymin

 Vasoactive intestinal peptide

Gastric inhibitory polypeptide

Somatostatin

Запитання 208

 Oral examination revealed dark yellow and brown spots and stripes on the labial and lingual surfaces of the patent's teeth, more than the half of the dental surface is affected; enamel and dentin are destroyed. What diagnosis is the most likely?

варіанти відповідей

Cuneiform defect

Fluorosis

Dental calculus

Dystrophic calcification.

Metastatic calcification

Запитання 209

 Longitudinal tooth section shoes a tissue that makes up the tooth basis and consists of collagen fibers, mineralized matrix, and tubules that hold dentinal fibers. This tissue develops from:

варіанти відповідей

External cells of enamel organ

Internal cells of enamel organ

Dental saccule

Intermediate cells of enamel organ

Peripheral part of dental papilla

Запитання 210

In the skin biopsy material in the epidermis there are cells with processes and cytoplasm that contains dark brown granules. Name these cells:

варіанти відповідей

Intraepidermal macrophages

Merkel's cells

Lymphocytes

Melanocytes

Keratinocytes

Запитання 211

 A 35-year-old woman is diagnosed with faucial diphtheria. The patient died with signs of acute heart failure. On autopsy: heart cavities are enlarged in the diameter, heart muscle is dull, flaccid, striped on section, with yellowish areas under the endocardium. What type of degeneration was detected in cardiac hystiocytes?

варіанти відповідей

Carbohydrate

Hyaline droplet

Ballooning

Fatty

Hydropic

Запитання 212

A 33-year-old man presents with disturbed pain and thermal sensitivity after a spinal cord trauma. The following ascending tract is injured:

варіанти відповідей

Spinothalamic

Ventral spinocerebellar

Anterior corticospinal

 Lateral corticospinal

Запитання 213

A patient with a bleeding knife wound in the area of carotid triangle has been delivered to a hospital. Blood flowing from the wound is dark. What vessel is injured?

варіанти відповідей

Internal jugular vein

External jugular vein

 Facial vein

Facial artery

Internal carotid artery

Запитання 214

In hot weather the bus passengers asked to open the roof hatches. What way of heat transfer is activated in this situation?

варіанти відповідей

Convection

Conduction and radiation

Conduction

Sweat evaporation

Radiation

Запитання 215

Enzymes and other active substances

regulating connective tissue density and

permeability are being produced in connective tissue cells. What enzyme drug is used to make the connective tissue growths looser and more permeable?

варіанти відповідей

Lipase

Lydase

Cocarboxylase

Cholinesterase

Amylase

Запитання 216

Rotenone is known to inhibit respiratory chain. What complex of mitochondrial respiratory chain is inhibited by this substance?

варіанти відповідей

Adenosine triphosphate synthetase

Cytochrome oxidase

Coenzyme Q- cytochrome c reductase

NADH-coenzyme Q reductase

Succinate-coenzyme Q reductase

Запитання 217

During examination of a patient with a periodontal disease it would be advisable to investigate functional state of blood vessels of the dentomaxillary area. What method can be applied in this case?

варіанти відповідей

Gnathodynamometry

Rheography

Chronaximetry

Electroodontodiagnostics

Sphygmography

Запитання 218

A patient suffers from an inflammatory process in the area of the lower wall of orbital cavity. What anatomical structure makes it possible for the inflammatory process to spread to the

pterygopalatine fossa?

варіанти відповідей

Optic canal

Inferior orbital fissure

 Superior orbital fissure

 Posterior ethmoidal foramen

Supraorbital incisure

Запитання 219

A patient with heatstroke was delivered to the admission room. What compensatory reactions develop in the

patient's body in such case?

варіанти відповідей

 Peripheral vasoconstriction

 Increased heart rate

 Coronary vasospasm

Peripherial vasodilatation

Persistent hyperglycemia

Запитання 220

At a certain stage of human ontogenesis, physiological bond occurs between circulatory systems of the mother and the fetus. This function is being carried out by the following provisory organ:

варіанти відповідей

Amnion

Placenta

Allantois

Serous tunic

Yolk sac

Запитання 221

For biochemical diagnostics of cardiac infarction it is necessary to determine activity of a number of enzymes and their izoenzymes in the blood. What enzyme assay is considered to be optimal for confirming or ruling out cardiac infarction at the early stage, after the patient develops thoracic pain?

варіанти відповідей

 LDH, isoenzymeisoenzyme isoenzymeisoenzyme sis

Cytoplasmic isoenzyme of aspartate aminotransferase

Creatine kinase MM isoenzyme

 LDH, isoenzymeisoenzyme isoenzymeisoenzyme si

Creatine kinase MB isoenzyme

Запитання 222

A 50-year-old man declined anaesthesia during dental manipulations. Due to severe pain he developed amuria caused by acute increase in production of:

варіанти відповідей

Thymosin

Glucagon

Renin

Thyroxin

Adrenaline

Запитання 223

During postmortem examination of a

9-month-old infant it was determined that the cause of death ras cerebral edema. What water-electrolyte imbalance is the most likely cause of the edema development?

Запитання 224

During postmortem examination of a

9-month-old infant it was determined that the cause of death ras cerebral edema. What water-electrolyte imbalance is the most likely cause of the edema development?

варіанти відповідей

Hyperosmolar hyperhydration

 Isoosmolar dehydration

Isoosmolar hyperhydration

Hypoosmolar hyperhydration

Hyperosmolar dehydration

Запитання 225

A patient presents with lymphocytic-monocytic leukogram pattern. It is characteristic of

варіанти відповідей

Acute inflammatory process

Allergies

Chronic inflammatory process

Chronic radiation sickness

-

Запитання 226

A patient with knife wound of the neck presents with hemorrhage. Initial wound management revealed damage to the vessel that is located along the lateral edge of the sternocleidomastoid muscle. Name this vessel:

варіанти відповідей

 A. carotis interna

A. carotis externa

V. jugularis anterior

V. jugularis interna

V. jugularis externa

Запитання 227

Electric current has affected skeletal muscle fiber resulting in depolarisation of the membrane. Depolarisation develops due to the following ions penetrating the membrane:

варіанти відповідей

2+

Na+

CH

H3-

K+

Запитання 228

To perform conduction anesthesia a patient had been administered a drug used in dental surgery. The patient developed the symptoms of poisoning: central nervous system excitation followed by paralysis and acute cardiovascular insufficiency (collapse). Additionally there were allergic reactions (itching, swelling, erythema).Name this drug.

варіанти відповідей

Thiopental sodium

Suxamethonium chloride

Pipecuronium bromide

Tubocurarin chloride

Lidocaine

Запитання 229

After anaesthetic application during tooth extraction the patient developed marked soft tissue edema of the upper and lower jaw, skin rash on the face, reddening, and itching. What pathological process results in such reaction to the anaesthetic?

варіанти відповідей

Toxic action of a drug

Disturbed lymph drainage

Circulatory deficiency

Allergy

Inflammation

Запитання 230

A man with signs of intestinal obstruction was delivered to a hospital. In the process of treatment, roundworms 25-40 cm in size were extracted from the patent's intestine. Determine the species of this helminth:

варіанти відповідей

Ascaris lumbricoides

 Ancylostoma duodenale

Strongyloides stercoralis

Trichocephalus trichiurus

Enterobius vermicularis

Запитання 231

Fetal malformations can be caused by such maternal diseases as rubella, syphilis, toxoplasmosis, cytomegaly, herpes, and chlamydiosis. These malformations belong to the following type of variability:

варіанти відповідей

Epimutational

Mutational

Combinative

Genomic imprinting

Modification

Запитання 232

What drugs are used for specific treatment of diphtheria?

варіанти відповідей

Placental gamma globulin

Native plasma

Antibiotics

Anatoxin

Antitoxic serum

Запитання 233

During examination of a child's oral cavity a dentist noted the appearance of the first permanent molars on the child's lower jaw. How old is the child

варіанти відповідей

6_7

10_11

 8_9

12_13

4_5

Запитання 234

A patient with chronic renal failure

presents with reduced imulin clearance of ml/min. The following renal function is disturbed:

варіанти відповідей

Reabsorption in the proximal tubular segment of the nephron

 Reabsorption in the tubules of collecting duct

Tubular secretion

Glomerular filtration

Reabsorption in the distal tubular segment of the nephron

Запитання 235

A. patient with symptoms of acute heart failure, namely pallor, acrocyanosis, and rapid shallow breathing, has been delivered to an emergency department. Which of these drugs is indicated in this case?

варіанти відповідей

Corglycon

Cordiamine

Nitroglycerine

 Digitoxin

Adrenalin hydrochloride

Запитання 236

A 47-year-old man consulted a dentist about difficult mouth opening (trismus). The patient has a history of a stab wound of the lower extremity. What infection can be manifested by these symptoms?

варіанти відповідей

Tetanus

Anaerobic wound infection

Brucellosis

Tularemia

Whooping cough

Запитання 237

An athlete (a long-distance runner) during competition has developed acute heart failure. This pathology developed due to

варіанти відповідей

Pressure overload

Volume overload

Pericardial pathology

Coronary blood flow disturbance

Direct damage to myocardium

Запитання 238

An experimet was conducted to study major indicators of hemodynamics. What hemodynamics indicator would be the same for both systemic and pulmonary circulation?

варіанти відповідей

Vascular resistance

 Mean arterial pressure

Linear blood flow velocity

Dyastolic blood pressure

Volumetric blood flow rate

Запитання 239

A patient, who after a trauma suffers from impeded active flexion of elbow, consulted a traumatologist. What muscle is the most likely to be damaged?

варіанти відповідей

M. deltoideus

M. biceps brachii

M. pectoralis major

M. latissimus dorsi

M. coracobrachialis

Запитання 240

To treat peptic ulcer disease of the stomach a patient has been prescribed famotidine. Specify the mechanism of action of this drug:

варіанти відповідей

Antienzyme action

Effect on cell membrane transport system

Physicochemical interaction

H2, histamine receptors block

Effect on ion channels of cell membranes

Запитання 241

A woman has come to a dentist with complaints of bruising and swelling around her eye. Anamnesis is as follows: several days prior her Ist premolar tooth had been extracted, with infraorbital anaesthesia administered; several days later hematoma appeared in the area of foramen intraorbitale. The branch of the following artery was damaged:

варіанти відповідей

 Superficial temporal artery

Maxillary artery

Facial artery

Superior labial artery

Masseteric artery

Запитання 242

A patient is registered for regular check-ups. Laboratory analyses for viral hepatitis diagnostics are made. In the blood serum only antibodies to HbsAg are detected. Such result is indicative of:

варіанти відповідей

Acute viral hepatitis type C

Viral hepatitis type A

Past case of viral hepatitis type B

 Chronic viral hepatitis type C

Acute viral hepatitis type B

Запитання 243

Microscopy of a plaque-like structure

extracted from the lateral surface of the tongue of a man with dentures revealed significant thickening of the epithelial layer along with processes of parakeratosis, hyperkeratosis, and acanthosis, in the connective tissue there are small round cell infiltrations. Make the diagnosis of the given pathological state:

варіанти відповідей

Chronic glossitis

Leukoplakia

Chronic stomatitis

Atrophic (Hunter's) glossitis

Ichthyosis

Запитання 244

A 69-year-old patient developed a small plaque with subsequent ulceration on the skin of the lower eyelid. The formation was removed. Microscopic examination of dermis revealed complexes of atypical epithelial cells arranged perpendicularly to the basal membrane on the periphery. The cells were dark, of polygonal prismatic shape, and had hyperchromic nuclei with frequent mitoses. What is the histological form of

carcinoma in this patient

варіанти відповідей

 Nonkeratinizing squamous cell carcinoma

Undifferentiated carcinoma

Keratinizing squamous cell carcinoma

Basal cell carcinoma

Adenocarcinoma

Запитання 245

 After the water supply system had been put into operation in a new residential area, the medical officers of sanitary and epidemiological station measured total microbial number in the water. Name the maximum permissible value of this indicator for potable water:

варіанти відповідей

1000

100

400

10

500

Запитання 246

In the course of an experiment researchers stimulate a branch of the sympathetic nerve that innervates heart. What alterations of cardiac activity should be registered?

варіанти відповідей

Increase in heart rate and heart force

Increase in heart rate

Increase in heart force

Decrease in heart force

Increase in arterial pressure

Запитання 247

2 days after a hunter cut a ground squirrel's body, he developed fever up to 90C, his lymph nodes enlarged. Later he developed pneumonia with serohemorrhagic exudate that contained egg-shaped microorganisms with bipolar staining. What provisional diagnosis can be made in this case?

варіанти відповідей

Brucellosis

Anthrax

Pseudotuberculosis

Plague

Tetanus

Запитання 248

Microscopy of an oval cell, 150 micron in size, revealed the following: cytoplasm has yolk inclusions but no centrioles. Name this cell:

варіанти відповідей

Leucocyte

Oocyte

 Macrophage

Fibroblast

Myocyte

Запитання 249

 A diabetus mellitus patient developed

unconsciousness and convulsions after

administration of insulin. What result of bloodglucose analysis is the most likely in this case?

варіанти відповідей

1.5 mmol/L

3.3 mmol/L

S mmol/L

10 mmo/L

 5.5 mmol/L

Запитання 250

 At the cementoenamel junction there are non-calcified areas, through which infection often penetrates into the tooth. Name these structures,

варіанти відповідей

Tomes' dentinal fiber

Odontoblasts

Enamel prisms

Ameloblasts

Enamel tufts

Запитання 251

A worker of a cattle farm consulted a surgeon about fever up to 0oC, headache, weakness. Objective examination of his back revealed hyperemia and a dark red infiltration up to 5 cm in diameter with black bottom in its center, which was surrounded with pustules. What disease are these presentations typical ooC, headache, weakness. Objective examination of his back revealed hyperemia and a dark red infiltration up to 5 cm in diameter with black bottom in its center, which was surrounded with pustules. What disease

are these presentations typical of?

варіанти відповідей

Anthrax

 Plague

Abscess

Tularemia

Furuncle

Запитання 252

A patient is diagnosed with inflammatory process in the area of the excretory duct of submandibular gland. This duct opens to:

варіанти відповідей

Caruncula sublingualis

Linea terminalis

Recesus gingivalis

Vestibulum oris

Foramen caecum linguae

Запитання 253

 Due to overdosage of a vasodilator a 58-year-old patient has developed acute vascular insufficiency. What drug would be advisable for termination of this pathological condition?

варіанти відповідей

Aethimizolum (Methylamide)

Cordiamin

Euphyllin (Aminophylline)

Mesaton (Phenylephrine)

Dopamine

Запитання 254

A girl presents with high fever and sore throat. Objectively the soft palate is swollen, the tonsils are covered with gray films that are firmly attached and leave deep bleeding lesions when removed. What is the most likely disease in this case?

варіанти відповідей

Necrotic tonsillitis

Infectious monomucl

Pseudomembranous (Vincent's) tonsillitis

Pharyngeal diphtheria

Lacunar tonsillitis

Запитання 255

Detoxification of bilirubin occurs in the membranes of endoplasmic reticulum of hepatocytes. Bilirubin is secreted by hepatocytes into bile for the most part as:

варіанти відповідей

Unconjugated bilirubin

Indirect reacting bilirubin

Bilirubin diglucuronide

Bilirubin monoglucuronide

Запитання 256

A 25-year-old man has lost all sensitivity due to damage of his peripheral nerves. Name this disorder.

варіанти відповідей

Ataxia

Hyperesthesia

Hypoesthesia

Anesthesia

Запитання 257

A patient on the 2nd day after cardiac infarction presents with acute decrease of systolic blood pressure down to 60 mm Hg with tachycardia 140/min., dyspnea, loss of consciousness. What mechanism is essential in the pathogenesis of shock developed in this case?

варіанти відповідей

Increased myocardial excitability caused by

products of necrotic disintegration

Development of anaphylactic reaction to

myocardial proteins

Development of paroxysmal tachycardia

Decreased circulating blood volume

Decreased cardiac output

Запитання 258

During tooth brushing it is not uncommon for oral unicosa to be injured. However, bleeding quickly stops on its own. What substances in saliva quickly staunch the flow of blood during minor oral injuries?

варіанти відповідей

Lipolytic enzymes

Amylolytic enzymes

Procoagulants

Mineral substances

Lysozyme and mucin

Запитання 259

A patient complains of urine excretion that occurs during sexual intercourse. What organ is affected?

варіанти відповідей

Urinary bladder

Seminal vesicles

Testicle

Epididymis

Prostate

Запитання 260

Chronic inflammation of gingiva resulted in excessive growth of connective tissue fibers. What cell elements are leading in the development of this condition?

варіанти відповідей

Fibroblasts

Osteoclasts

Fibrocytes

Osteoblasts

Macrophages

Запитання 261

A. patient has developed a painful sore with soft ragged edges in the oral cavity. Microscopically there is lymphocytic infiltration with epithelioid and giant multinucleate Langhans cells in the sore floor. What diagnosis is the most likely?

варіанти відповідей

Gangrenous stomatitis

Syphilis

Ulcer-cancer

Tuberculosis

Ulcerative stomatitis (Vincent's stomatitis)

Запитання 262

A patient suffering from malaria has developed hemolytic anemia after taking primaquine antimalarial drug. Hereditary insufficiency of the following enzyme in erythrocytes will be observed in this case:

варіанти відповідей

Fructose 1-phosphate aldolase

 Phosphofructokinase

Triosephosphate isomerase

Glucose 6-phosphate dehydrogenase

Lipase

Запитання 263

To prevent wound infection associated with surgical procedures a patient was given a synthetic antiprotozoan drug demonstrating high activity against Helicobacter pylori. Specify this drug:

варіанти відповідей

Chingamin

 Isoniazid

Acyclovir

Doxycycline hydrochloride

Metronidazole

Запитання 264

A surgeon must amputate the damaged part of the patient's foot along the line of Lisfranc joint. What ligament must be cut in this case?

варіанти відповідей

Medial interosseous tarsometatarsal ligament

Talonavicular ligament

 Talocalcaneal ligament

Calcaneonavicular ligament

Bifurcated ligament

Запитання 265

A 45-year-old patient with essential hypertension, who has been taking an antihypertensive drug for 4 days, notes that his blood pressure is normalized; however the patient complains of sleepiness and sluggishness. What drug is the patient talking?

варіанти відповідей

Prazosin

Captopril

Apressin (Hydralazine)

Clophelin (Clonidine)

Enalapril

Запитання 266

Mucin aggregates retain water, which

results in their viscosity and protective action. It is possible because mucin structure contains:

варіанти відповідей

 Glucose

Homopolysaccharides

Disaccharides

Oligosaccharides

Glycosaminoglycans

Запитання 267

A. 20-year-old woman came to the doctor with complaints of general weight loss, loss of appetite, weakness, skin discoloration resembling bronze tan. In addtition to hyperpigmentation, examination in the hospital revealed bilateral adrenal tuberculosis. What substance leads to skinhyperpigmentation, excessively? when accumulated

варіанти відповідей

Hemozoin

Lipofuscin

Melanin

Bilirubin

Adrenochrome

Obstructive

Запитання 268

Histologic specimen of renal cortex shows renal corpuscle and renal tubules. It is known that reabsorption of substances occurs in the renal rubules. What nephron tissue takes part in this process?

варіанти відповідей

Reticular tissue

Cartilaginous tissue

Mucous tissue

Connective tissue proper

Epithelial tissue

Запитання 269

 Psychological evaluation determined that a person is able to quickly adapt to changing situation, has good memory, is emotionally stable, possesses of high working ability. This person is the most likely to be:

варіанти відповідей

Choleric

Sanguine

Phlegmatic

Melancholic

Phlegmatic with melancholic traits

Запитання 270

One of the coats of a hollow organ has anastomotic fibers with nuclei. The fibers consist of cells that form intercalated disks at the places of contact. What tissue forms this coar?

варіанти відповідей

Cross-striated skeletal muscle

Dense irregular connective tissue

 Loose fibrous connective tissue

Cross-striated cardiac muscle

Unstriped muscle

Запитання 271

Miners' work at the coal-face often leads to development of anthracosis. What type of respiratory failure arises along with this disease?

варіанти відповідей

Thoracic

Diaphragmatic

 Dysregulatory

Restrictive

Запитання 272

A 12-year-old boy has tetanic convulsions. What gland can be functionally impaired in this case?

варіанти відповідей

Glandula thyroidea

Glandulae parathyroidae

Thymus

Hypophisis

Glandula pinealis

Запитання 273

A patient with megaloblastic anemia was taking a water-soluble vitamin. Name this substance:

варіанти відповідей

Thiamine chloride

Pyridoxine

Cyanocobalamin

 Tocopherol acetate

Ascorbic acid

Запитання 274

Autosomal nondisjunction had occurred in a woman during meiosis. An ovum with the third copy of the 18th chromosome was formed. The ovum was impregnated by normal spermatozoon.

The resulting child will suffer from:

варіанти відповідей

Down's syndrome

Patau's syndrome

Klinefelter's syndrome

Edwards' syndrome

Запитання 275

A patient developed burning sensation in the oral cavity and white fuzzy coating on the tongue. Oral thrush is diagnosed. What drug of those listed below should be used?

варіанти відповідей

Griseofulvin

Tetracycline

Amphotericin

Gentamicin

Nystatin

Запитання 276

Several patients with similar complaints came to the doctor. They all present with weakness, pain in the intestines, indigestion. Feces analysis revealed the need for urgent hospitalization of the

patient, who had microbial cysts with four nuclei detected in his samples. Such cysts are characteristic of the following protozoon:

варіанти відповідей

Trichomonad

Entamoeba coli

Entamoeba histolytica

Balantidium

Lamblia

Запитання 277

 Lateral X-ray of the occipital bone area demonstrates fracture of the occipital condyle. Integrity of the following anatomical structure is disturbed:

варіанти відповідей

 Foramen stylomastoideum

Foramen mastoideum

Canalis caroticus

Canalis nervi hypoglossi

Foramen ovale

Запитання 278

A patient undergoing treatment with nitrofurans has imbibed a small amount of alcohol, which resulted in severe poisoning. The poisoning developed due to:

варіанти відповідей

Neural disorder

Allergic reaction

Disturbed renal function

Accumulation of acetic aldehyde

Cardiovascular collapse

Запитання 279

In what organ biotransformation (metabolic transformation) of most medicinal agents occurs upon their introduction into an organism?

варіанти відповідей

 Lungs

Skin

Liver

Intestine

Kidneys

Запитання 280

A 40-year-old patient suffers from intolerance of dairy products. This condition has likely developed due to insufficiency of the following digestive enzyme:

варіанти відповідей

Amylase

Lipase

Lactase

Maltase

Invertase

Запитання 281

A patient with ischemic heart disease presents with increased blood plasma content of triglycerides and very lowdensity lipoproteins.. What drug should be prescribed?

варіанти відповідей

Fenofibrate

Famotidine

Dobutamine

Amiodarone

Lisinopril

Запитання 282

An experiment was conducted to measure the threshold of tactile receptors stimulation with various stimuli. What stimulus will have the lowest threshold?

варіанти відповідей

Chemical stimulus

Mechanical stimulus

 Heat stimulus

Photic stimulus

Cold stimulus

Запитання 283

ormal metabolic rate and energy expenditure should be taken into account when actual basal metabolic rate of a patient is being determied by means of indirect calorimetry. Normal metabolic rate can be determined most accurately based on the patient's:

варіанти відповідей

Respiratory coefficient and caloric coefficient of oxygen

Height and respiratory coefficient

Sex, age, height and weight

 Respiratory coefficient and body surface area

Body surface area and weight

Запитання 284

Due to severe pain syndrome a patient has been prescribed a narcotic analgesic. Specify the prescribed drug

варіанти відповідей

Dimexid

Morphine

Analgin (Metamizole)

Nimesulid

Indometacin

Запитання 285

A. 49-year-old man presents with facial edema, significant proteinuria, hypoproteinemia, dysproteinemia, and hyperlipidemia. What provisional diagnosis can be made?

варіанти відповідей

Cystitis

 Pyelonephritis

Prostatitis

Urolithiasis

Nephrotic syndrome

Запитання 286

A patient with acute bronchitis has been prescribed sulfanilamide drugs for treatment. In an hour after administration the patient developed itching and vesicles filled with light transparent liquid on the face, palms and soles. Name the mechanism of immune response:

варіанти відповідей

 Antibody-mediated cytolysis

Reaginic reaction

Cell cytotoxicity

Immune complex-mediated hypersensitivity

Запитання 287

Oral examination reveals marked reddening of mucosa at the root of the tongue. What structure is involved in the inflammatory process?

варіанти відповідей

Veil of palate

Tonsil of torus tubaris

Pharyngeal tonsil

Lingual tonsil

 Palatine tonsil

Запитання 288

 During ultrasound a patient with atherosclerosis was diagnosed with bilateral stenosis of the renal arteries. Specify the bioactive substance that is the key pathogenetic link in the development of arterial hypertension in this case:

варіанти відповідей

Thyroxin

Adrenaline

Vasopressin

Cortisol

Renin

Запитання 289

A patient, who had been eating only polished rice, developed polyneuritis caused by thiamine deficiency. What compound can be indicative of this kind of avitaminosis when excreted with urine?

варіанти відповідей

Pyruvic acid

Uric acid

Phenylpyruvate

Methylmalonic acid

Malate

Запитання 290

In the epicenter of the registered rabies cases among wild animals a 43year-old man came to a clinic and claimed to have been bitten by a stray dog. He was given a course of anti-rabies vaccine. This preparation belongs to the following type of vaccines:

варіанти відповідей

Toxoids

Attenuated

Synthetic

Molecular

Inactivated

Запитання 291

A man has developed downturning mouth and smoothed out nasolabial fold due to influenza complication. What nerve is damaged?

варіанти відповідей

Facial nerve

Maxillary nerve

Oculomotor nerve

Mandibular nerve

Trochlear nerve

Запитання 292

To assess the rate of collagen

disintegration during certain connective tissue disturbances, it is necessary to measure the urine content of the following:

варіанти відповідей

Oxyproline

Proline

Ornithine

Lysine

 Urea

Запитання 293

Dopamine precursor-dioxyphenylalanine (DOPA) is used in treatment of Parkinson's disease. This active substance is produced from the

following amino acid:

варіанти відповідей

Histidine

Tryptophan

Tyrosine

Cysteine

Alanine

Запитання 294

A patient was diagnosed with Klinefelter's syndrome. The patient with this disease will have the karyotype (47, XXY). How many sex chromosomes are in this complement?

варіанти відповідей

Τwo

Three

One

Zero

Forty four

Запитання 295

A woman is diagnosed with fatty tissue inflammation located between the leaves of broad ligament of the uterus. Name this anatomical structure:

варіанти відповідей

Myometrium

Endometrium

Perimetrium

Mesometrium

Parametrium

Запитання 296

 In a 12-year-old patient an inflammatory process in the internal ear spread to the meninges, diffusely affecting them. A doctor suspects the process to have spread through the connection between the subarachnoid space of the brain and perilymphatic space of the intermal ear. What anatomic structure became the pathway for the spreading inflammation?

варіанти відповідей

Hiatus canalis n petrosi minoris

Hiatus canalis n. petrosi majoris

Fossa subarcuata

Fissura petrosquamosa

Aqueductus vestibuli

Запитання 297

 A patient developed a tender red nodule in the lower jaw area. Histologically there is accumulation of purulent exudate in several hair follicles. What clinicopathological type of inflammation is observed?

варіанти відповідей

 Furuncle

Carbuncle

Abscess

Hypostatic abscess

Phlegmon

Запитання 298

A patient with diabetes mellitus has been delivered to a hospital unconscious. BP is low, Kussmaul's respiration is observed, the smell of acetone can be detected from the patient's mouth. What mechanism is leading in the coma development in this case?

варіанти відповідей

Accumulation of sodium ions

Accumulation of ketone bodies in blood

Accumulation of potassium ions

Accumulation of urea

Accumulation of chlorine ions

Запитання 299

Resuscitation unit received a patient with acute poisoning caused by unidentified medicine. To quickly excrete the poison from the patient's body, forced diuresis was induced. What substance was used to perform this procedure?

варіанти відповідей

Omeprazole

 Dithylinum (Suxamethonium chloride)

Hydrochlorothiazide

Spironolactone

Furosemide

Запитання 300

High-altinide dwellers Typically demonstrate chronically intensified respiration and decreased pCO2 value of blood. What mechanism is leading in the compensation of their asid-base imbalance?

варіанти відповідей

Decreased pulmonary ventilation

Increased pulmonary ventilation

Increased ammonia excretion with urine

Desreased renal reabsorption of bicarbonate

Запитання 301

A 16-year-old girl, who has been starving herself for a long time to lose weight, developed an edema. This phenomenon is mainly caused by:

варіанти відповідей

Hypoproteinemia due to protein synthesis disturbance

Decreased production of vasopressin in the hypothalamus

Deceleration of glomerular filtration rate

Hypoglycemia due to glycogen synthesis

Venous congestion and increased venous

pressure

Запитання 302

. A 30-year-old man with glomerulonephritis has developed nephrotic syndrome. What symptom invariably accompanies nephrotic syndrome?

варіанти відповідей

Low urine specific gravity

Glucosuria

Proteinuria

Azotemia

Anemia

Запитання 303

A patient with skin burns was delivered to a hospital. To clean the wound from necrotic tissues and mucus the doctor prescribed an enzymatic drug for topical treatment. Name this drug

варіанти відповідей

Streptokinase

Tripsin

Asparaginase

Pancreatin

Pepsin

Запитання 304

During examination of a woman she was found to have a luminal narrowing of the right jugular foramen (foramen jugulare). What cranial bones form this foramen?

варіанти відповідей

Occipital and frontal

Cuneiform and palatine

Occipital and cuneiform

Temporal and cuneiform

Temporal and occipital

Запитання 305

 A hunter was drinking raw water from a pond. He risks infection with the following type of trematodiasis:

варіанти відповідей

Opisthorchiasis

Clonorchiasis

Fascioliasis

Dicroceliasis

Paragonimiasis

Запитання 306

. Dentists have high risk of contracting viral hepatitis type B in the course of their duties and therefore are subject to mandatory vaccination What vaccine is used in such cases?

варіанти відповідей

Inactivated vaccine

Chemical vaccine

Recombinant vaccine

Anatoxin

Live vaccine

Запитання 307

 In Western Europe nearly half of all congenital malformations occur in the children conceived in the period when pesticides were used extensively in the region. These congenital conditions are the result of the following factor:

варіанти відповідей

Carcinogenic

Malignization

Mechanical

Teratogenic

Mutagenic

Запитання 308

 There is increased activity of AST, LDH, LDH and CPK in the patient's blood. Pathological process most likely occurs in the:

варіанти відповідей

Heart

 Skeletal muscles

Adrenal glands

 Liver

Kidneys

Запитання 309

During investigation of skin epithelium it was determed, that it consists of several cellular layers. Epithelial cells of the external layer have по nuclei. What type of epithelium is it?

варіанти відповідей

Multirow ciliary epithelium

Multirow columnar epithelium

Keratinized stratified squamous epithelium

Transitional epithelium

Non-keratinized stratified squamous epithelium

Запитання 310

Cytogenetic analysis allowed to determine the patient's karyotype-47, XY, +21/46, XY. Name this condition:

варіанти відповідей

Phenocopy

Deletion

Mosaicism

Genocopy

Translocation

Запитання 311

During acute inflammation of parotid

gland, there is damage to the cells of secretory segments observed. What cells are damaged in this case?

варіанти відповідей

Seromucous cells

Albuminous cells, serous cells, mucous cells

Serous cells, cells with basal striation, stellate cells

Brush-bordered epithelial cells, cells with basal striation

Serous cells, myoepithelial cells

Запитання 312

Autopsy of a man, who died suddenly with signs of acutely disturbed cerebral circulation, revealed aneurysm rupture of the medial cerebral artery and a round cavity 4 cm in diameter filled with blood in his frontal lobe. Name this type of

hemorrhage:

варіанти відповідей

 Contusion

 Hemorrhagic infiltration

 Petechiae

Hematoma

Запитання 313

A 30-year-old breastfeeding woman keeps to the diet that daily provides her with 1000 mg of calcium, 1300 mg of phosphorus, and 20 mg of iron. How should the daily dosages of minerals in this diet be adjusted?

варіанти відповідей

Increase calcium intake

Decrease fluorine intake

Decrease iron intake

Increase phosphorus intake

Increase iron intake

Запитання 314

A 13-year-old girl is an in-patient at the hematology department of the regional children's hospital. She was diagnosed with iron-deficiency anemia. What type of hypoxia does this patient have?

варіанти відповідей

Hemic

Circulatory

Mixed

Respiratory

Tissue

Запитання 315

A newborn boy has been diagnosed with hydrocephalus. Doctors consider it to be caused by teratogenic factors. What germ layers are affected by the teratogen?

варіанти відповідей

Endoderm

Endoderm and mesoderm

Ectoderm

Mesoderm

All embryo germ layers

Запитання 316

Premature babies often develop respiratory distress syndrome. This pathology is caused by the deficiency of a certain component of the blood-air barrier. Name this component:

варіанти відповідей

Endothelial basement membrane

Alveolar basement membrane

Capillary endothelium

Surfactant

Запитання 317

A patient has deep lacerated wound with uneven edges. The wound is suppurating; its edges present with moist granulation tissue that does not protrude above the wound level. Name the type of

wound healing:

варіанти відповідей

Healing by secondary intention

 Healing by primary intention

Direct closure of the epithelial defect

Wound organization

Healing under the scab

Запитання 318

Deaf parents with genotypes DDee and

ddEE gave birth to a girl with normal hearing.Specify the form of D and E genes interaction:

варіанти відповідей

Epistasis

Complementary interaction

Polymery

Overdominance

Complete dominance

Запитання 319

To treat chronic heart failure a patient

takes digoxin. What diuretic can increase digoxin toxicity due to increased excretion of K+ions?

варіанти відповідей

Silibor

Lisinopril

Panangin

Spironolactone

Hydrochlorothiazide

Запитання 320

A 20-year-old young man, who started to train systematically in athletics, has the following resting-state blood values: erythrocytes 5,5- 12/1, reticulocytes 12%, hemoglobin 160 g/l, color index 1,03. Such blood values indicate erythropoiesis stimulation due to the following occurring in the process of his training:

варіанти відповідей

Hyperventilation

Physical activity

Hypercapnia

Hypoxemia

Hyperglycemia

Запитання 321

Histologic specimen demonstrates an oral cavity organ with mucosa covered with keratinized stratified squamous epithelium. Specify this organ or its part:

варіанти відповідей

Gum

 Uvula

Labial mucosa

Soft palate

Inferior surface of tongue

Запитання 322

 Cytochemical investigation revealed high content of hydrolytic enzymes in the cytoplasm This phenomenon indicates the activity of the following organelles:

варіанти відповідей

Mitochondria

Lysosomes

Cell center

Polysomes

 Endoplasmic reticulum

Запитання 323

At the crown apex of the second molar, on the surface that comes into contacts with the cheek, the doctor detected a carious cavity. Name the affected crown surface:

варіанти відповідей

Facies distalis

Facies occlusalis

Facies vestibularis

Facies lingualis

Facies mesialis

Запитання 324

A patient presents with damaged fibers of the ninth pair of cranial nerves (glossopharyngeal nerve). What gustatory sensation will be disturbed in this case?

варіанти відповідей

Bittermess

 All gustatory sensations

Sourmess

Saltiness

Sweetness

Запитання 325

In a closed community it is necessary to determine community members immunity to diphtheria and verify the need for their vaccination What investigation is necessary in this case?

варіанти відповідей

Check medical records for vaccination

 Determine diphtheria antibody titer

Test community members for diphtheria bacillus carriage

Determine antitoxin titer by means of indirect hemagglutination assay

Determine community members immunity to diphtheria bacillus

Запитання 326

A patient with maxillofacial joint arthritis has come to a dentist. The dentist prescribed an ointment with an antiinflammatory agent that is a pyrazolone derivative. Name this agent:

варіанти відповідей

Diclofenac sodium

Ibuprofen

Mefenamic acid

Butadion (Phenylbutazone)

 Indometacin

Запитання 327

 Due to trauma the patient's parathyroid glands have been removed, which resulted in inertness, thirst, sharp increase of neuromuscular excitability. Metabolism of the following substance is disturbed:

варіанти відповідей

Manganese

Calcium

Zinc

Chlorine

Molybdenum

Запитання 328

After inhalation of dust a person develops cough, which results from stimulation of:

варіанти відповідей

Irritant receptors

Nociceptors

Pulmonary thermoreceptors

Juxtacapillary receptors

Pulmonary chemoreceptors

Запитання 329

.Laboratory analysis UDPglucuronyl transferase deficiency in the patient. What blood values can confirm this enzymopathy?

варіанти відповідей

Uremia

Phenylketonuria

Ketoacidosis

Hyperbilirubinemia

Indicanuria

Запитання 330

Holocrine secretion is characteristic of sebaceous glands. What structural components ensure renewal of the cells of these glands?

варіанти відповідей

Germinative layer cells

Myoepithelial cells

Stratified squamous epithelium of the excretory duct

Sebocytes

 Nonstratified cuboidal epithelium of the excretory duct

Запитання 331

An oculist has detected increased time of darkness adaptation of a patient's eye. What vitamin deficiency can cause such symptom?

варіанти відповідей

K

C

A

S

Запитання 332

Microphotogram made with electron microscope shows alveolar cells that compose blood-air barrier. Name this cells:

варіанти відповідей

Villous epithelial cells

Alveolar respiratory epithelial cells

Clara cells (club cells)

Alveolar macrophages

Alveolar secretory epithelial cells

Запитання 333

 Histological investigation of renal biopsy material taken from a patient with tuberculosis has revealed chaotically located chromatin granules in the focus of caseous necrosis. These changes are the result of:

варіанти відповідей

Karyorrhexis

Karyopyknosis

 Karyolysis

Apoptosis

Mitotic activity of nuclei

Запитання 334

 Significant shortcoming of microscopy in infection diagnostics is its insufficient information value due to morphological similarity between many species of microorganisms. What immunoassay can significantly Increase informativity of this method?

варіанти відповідей

Radioimmunoassay

Fluorescence immunoassay

Coombs' test

Opsonization

mmune-enzyme assay

Запитання 335

 For two weeks a woman has been taking the mixture for neurasthenia, which was prescribed by a neurologist. Her general state slightly improved but shortly she started complaining of rhinitis, conjunctivitis, skin rashes, fatigue, and

memory impairment. What group of drugs can have such a side effect?

варіанти відповідей

Hop preparations

Adaptogens

Bromine salts

Valerian preparations

 Motherwort preparations

Запитання 336

A 50-year-old man came to a hospital

with complaints of memory disorders, painful sensations along the nerve trunks, decreased mental ability, circulatory disorders and dyspepsia. Anamnesis states excessive alcohol consumption.

What vitamin deficiency can result in such

symptoms?

варіанти відповідей

Thiamine

 Riboflavin

Calciferol

 Niacin

Retinol

Запитання 337

 The patient's saliva has been tested for antibacterial activity. What saliva component has antibacterial properties?

варіанти відповідей

Lysozyme

 Ceruloplasmin

Cholesterol

Parotin

 Amylase

Запитання 338

A girl is diagnosed with primary microcephaly that is a monogenic autosomal recessive disease. Her natural brother develops normally. What genotypes do the parents of these children have?

варіанти відповідей

aa x aa

AABB x AABB

Aa x Aa

AA x AA

vAA x an

Запитання 339

A patient during fasting developed ketoacidosis as a result of increased fatty acids decomposition. This decomposition can be inhibited with

варіанти відповідей

Adrenaline

Thyroxin

Cortisol

Insulin

Glucagon

Запитання 340

A person found oneself in an emotionally straining situation. As the result the blood adrenaline level has risen, therefore increasing the strength of cardiac contractions. In what way does adrenaline increase the strength of cardiac contractions?

варіанти відповідей

ctivates peripheral chemoreceptors

Decreases tone of vagus nerves

Activates cardiac B-adrenergic receptors

Decreases excitability of pacemaker cells

 Activates vascular baroreceptors

Запитання 341

A patient has developed a grayish-white dense protruding focus on the oral mucosa. Histologycally there are hyperkeratosis, parakeratosis, and acanthosis of epithelium with lymphocytic and plasmocytic infiltration of underlaying connective tissue in this area. What pathology has developed in the oral mucosa?

варіанти відповідей

Leukoplakia

Leukoderma

Local tumor-like amyloidosis

 Hyalinosis

Запитання 342

What factor results in the highest energy expenditure under the nommal vital activity conditions?

варіанти відповідей

Decrease of environment temperature

Action of skeletal muscles

Mental work

Food rich in calories

Increase of environment temperature

Запитання 343

It is necessary to decrease pumping ability of the patient's heart. What membrane cytoreceptors must be blocked to achieve this effect?

варіанти відповідей

a-adrenergic receptors

a- and B-adrenergic receptors

Nicotinic acetylcholine receptors

 Muscarinic acetylcholine receptors

B-adrenergic receptors

Запитання 344

During an experiment aimed as study of respiration regulation processes the peripheral chemoreceptors of test animals were stimulated,which resulted in changed respiratory rate and depth. Where are these receptors localized?

варіанти відповідей

Atria, carotid sinus

Capillary bed, aortic arch

Capillary bed, aortic arch, carotid simus

 Capillary bed, carotid sinus

Aortic arch, carotid simus

Запитання 345

 Electronic microscopy of a kidney shows tubules paved with cuboidal epithelium. In the epithelium there are light and dark cells. The light cells contain few organelles, their cytoplasm forms

folds. These cells provide reabsorption of wate from primary urine into blood. The dark cells structurally and functionally resemble gastric parietal cells. What tubules are shown on the microslide?

варіанти відповідей

Distal tubules

Proximal tubules

Ascending limb of loop of Henle

Descending limb of loop of Henle

Collecting tubules

Запитання 346

 A patient complains of toothache. On examination he has been diagnosed with pulpitis. Which factor played the main pathogenic role in the development of pain syndrome in this case?

варіанти відповідей

Inadequate stimulation of the mandibular nerve branch

) Vasospasm

Interleukin action

Increased intratissular pressure in the dental pulp

 Activation of one of the complement system components

Запитання 347

 42-year-old woman, who has been keeping to a vegetarian diet for a long period of time, consulted a doctor. Examination revealed negative nitrogen balance in the patient. What factor is the most likely cause of such a condition?

варіанти відповідей

Decreased rate of metabolic processes

Insufficient amount of proteins in the diet

Insufficient amount of fats in the diet

Insufficient amount of dietary fiber

Excessive amount of fats in the diet

Запитання 348

 Macroscopic examination of lung tissue revealed areas of high airiness with small bubbles. Histological examination revealed thinning and rupture of alveolar septa accompanied by formation of large diversiform cavities. What disease was

revealed in the lung?

варіанти відповідей

Chronic bronchitis

Cavernous tuberculosis

Pulmonary emphysema

Multiple bronchiectasis

 Fibrosing alveolitis

Запитання 349

A patient presents with high content of vasopressin (antidiuretic hormone) in the blood. What changes in the patient's diuresis will occur?



варіанти відповідей

Anuria

Oliguria

Polyuria

Glycosuria

Natriuria

Запитання 350

A 50-year-old man came to a hospital with complaints of memory disorders, painful sensations along the nerve trunks, decreased mental ability, circulatory disorders and dyspepsia. Anamnesis states excessive alcohol consumption What vitamin deficiency can result in such symptoms?

варіанти відповідей

Riboflavin

Thiamine

Retinol

Calciferol

Niacin

Запитання 351

A patient suffering from arthritis is prescribed a COX selective inhibitor with anti-inflammatory action. Specify this drug

варіанти відповідей

Celecoxib

Butadion (Phenylbutazone)

Analgin (Metamizole)

Indometacin

Dimexid

Запитання 352

 Several hours after the dental trauma the tooth pulp presents with hyperemic vessels, marked tissue edema with isolated neutrophils, lymphocytes, and minor dystrophic changes of nerve fibers. Make the diagnosis:

варіанти відповідей

Serous pulpitis

Fibrous pulpitis

Gangrenous pulpitis

 Granulating pulpitis

Suppurative pulpitis

Запитання 353

A patient with bronchial asthma has

developed a bronchial spasm during the visit to a dentist. Name the drug necessary to arrest the spasm:

варіанти відповідей

Naphthizin

Bisoprolol

Salbutamol

Anaprilin (Propranolol)

Mesaton (Phenylephrine)

Запитання 354

A 56-year-old man presents with parathyroid rumor. The following is observed: muscle weakness, osteoporosis, bone deformation, nephroliths consisting of oxalates and phosphates.

The patient's condition is caused by:

варіанти відповідей

Decreased secretion of parathyroid hormone

Increased secretion of calcitonin

Decreased secretion of calcitriol

Increased secretion of parathyroid hormone

Increased secretion of thyroxin

Запитання 355

A schizophrenic patient has been prescribed aminazine. What pharmacodynamic action of this drug justifies its prescription in this case?

варіанти відповідей

Antiemetic

 Hypotensive

Muscle relaxant

Antipsychotic

Hypothermic

Запитання 356

 A patient has been diagnosed with severe B12-deficient anemia with hemopoiesis disturbance. Anamnesis states total gastrectomy. What cells allow to confirm this diagnosis, if they are absent in the peripheral blood?

варіанти відповідей

Anulocytes

Megalocytes

Normocytes

Microcytes

Ovalocytes

Запитання 357

 In the patient's feces there were eggs of Fasciola hepatica. The doctor, however, refrained from making diagnosis and insisted on a repeat of analysis, with beef liver excluded from the patient's diet. What led the doctor to make such decision?

варіанти відповідей

Insufficient qualification of a laboratory assistant

Lack of trust towards the investigation method

Absent symptoms of invasion

Possible phenomenon of transient eggs

Uncertainty regarding the analysis precision

Запитання 358

 A dentist prescribed the patient with maxillofacial arthritis diclofenac sodium. What is the mechanism of action of this drug?

варіанти відповідей

Catalase inhibition

Phosphodiesterase activation

 Opiate receptors activation

Cyclooxygenase-2 inhibition

Opiate receptors block

Запитання 359

During their expedition to the Middle East, the students found a 7-centimeterlong arthropod. Its body consists of cephalothorax with 4 pairs of ambulatory legs and segmented abdomen with two venom glands in its last segment. The gland openings are located on the tip of the hookshaped sting. The animal was identified as a nocturnal predator, its venom is dangerous for humans. It belongs to the following order.

варіанти відповідей

Aranei

 Solpugae

 Acarina

Aphaniptera

Scorpiones

Запитання 360

A tumor is detected in one of the regions of the patient's brain, resulting in the patient's inability to maintain normal body temperature. What brain structure is damaged?

варіанти відповідей

 Striatum

 Cerebellum

Thalamus

Hypothalamus

Substantia nigra

Запитання 361

A doctor examined a patient, studied the patient's blood analyses and concluded that the peripheral organs of immunogenesis are damaged. What organs are the most likely to be affected?

варіанти відповідей

Yellow bone marrow

Red bone marrow

Kidneys

Thymus

Tonsils

Запитання 362

The autopsy of a 45-year-old man, who worked in organic acids production and died from uremia, has revealed reduced and completely destroyed crowns of the upper and lower incisors Microscopy detects destruction of dentin and enamel, the pulp is covered with dense replacing dentin. What pathological process occurred in the teeth in this case?

варіанти відповідей

Necrosis of the hard tooth tissues

 Teeth erosion

Hypercementosis

Fluorosis

Cuneiform defects

Запитання 363

 A 36-year-old patient consulted an ophthalmologist about eye ache. The examination revealed erosion of the cornea, that is the lack of superficial and spinous layers of the epithelium. What cells will provide regeneration of the


damaged epithelium?

варіанти відповідей

 Cells of stratum comeum

Cells of stratum superficiale

Cells of straum granulosum

Cells of stratum basale

Запитання 364

An ophthalmologist suspects blennorrhea (gonococcal conjunctivitis) in a child with signs of suppurative keratoconjunctivitis. What laboratory diagnostics should be conducted to confirm the diagnosis?

варіанти відповідей

Microscopy and bacteriological analysis

Serum diagnostics and allergy test

Biological analysis and phagodiagnostics

 Biological analysis and allergy test

Microscopy and serum diagnostics

Запитання 365

A 40-year-old woman is being treated at the therapeutics department. Her temperature chart shows cyclic fevers alternating with periods of temperature normalization that last for several days. What type of temperature profile is it?

варіанти відповідей

Febris intermittent

Febris continua

Febris recurrens

ebris remittens

Запитання 366

. Brain investigation by means of nuclear magnetic resonance revealed the patient to have a hematoma in the genu of the intermal capsule. What pathway is damaged in this case?

варіанти відповідей

Tr. cortico-nuclearis

Tr. cortico-spinalis

Tr. cortico-thalamicus

Tr. cortico-fronto-pontinus

Tr. thalamo-corticalis

Створюйте онлайн-тести
для контролю знань і залучення учнів
до активної роботи у класі та вдома

Створити тест